Download as pdf or txt
Download as pdf or txt
You are on page 1of 59

https://www.pdfnotes.

co << Download From >> https://

VISION IAS
www.pdfnotes.co

www.visionias.in

Test -4
Geography-II
Geography - II

Economic and Human Geography World-

▪ 12th NCERT: Geography India People


Economic Geography of the world:
-

and Economy
▪ primary, secondary, tertiary, quaternary ▪ 12th NCERT: Fundamental of Human
▪ and quinary activities Geography
Human Geography of world
World Population, Distribution & Density, ▪ 10th NCERT: Contemporary India
Part II
-

▪ Races & Tribes, Settlement & Migration ▪ Certificate Physical and Human
Economic
▪ and Human Geography - India
Agriculture Geography Goh Cheng Leong

-

▪ Orient Longman Atlas or Oxford Atlas


▪ Land Resources ▪ Vision IAS Value Addition Material-
▪ Water Resources
Eco. Geography
▪ Mineral and Energy Resources
▪ Industries
▪ Transport and Communication
Foreign Trade
Population, Migration, Settlements

PT - 2022 Google it:- https://www.pdfnotes.co


https://www.pdfnotes.co << Download From >> https://
www.pdfnotes.co

VISIONIAS
www.visionias.in

Test Booklet Series

TEST BOOKLET

GENERAL STUDIES (P) 2022 – Test–3473


C
Time Allowed: Two Hours Maximum Marks: 200

INSTRUCTIONS

1. IMMEDIATELY AFTER THE COMMENCEMENT OF THE EXAMINATION, YOU SHOULD CHECK THAT THIS BOOKLET
DOES NOT HAVE ANY UNPRINTED OR TURN OR MISSING PAGES OR ITEMS, ETC. IF SO, GET IT REPLACED BY A
COMPLETE TEST BOOKLET.

2. ENCODE CLEARLY THE TEST BOOKLET SERIES A, B, C OR D AS THE CASE MAY BE IN THE APPROPRIATE PLACE IN
THE ANSWER SHEET.

3. You have to enter your Roll Number on the Test Booklet in the Box
provided alongside. Do NOT write anything else on the Test Booklet.

4. This Test Booklet contains 100 items (Questions). Each item is printed in English. Each item comprises four
responses (answers). You will select the response which you want to mark on the Answer Sheet. In case you
feel that there is more than one correct response with you consider the best. In any case, choose ONLY ONE
response for each item.

5. You have to mark all your responses ONLY on the separate Answer Sheet provided. See direction in the
answers sheet.

6. All items carry equal marks. Attempt all items. Your total marks will depend only on the number of correct
responses marked by you in the answer sheet. For every incorrect response 1/3rdof the allotted marks will be
deducted.

7. Before you proceed to mark in the Answer sheet the response to various items in the Test booklet, you have to
fill in some particulars in the answer sheets as per instruction sent to you with your Admission Certificate.

8. After you have completed filling in all responses on the answer sheet and the examination has concluded, you
should hand over to Invigilator only the answer sheet. You are permitted to take away with you the Test
Booklet.

9. Sheet for rough work are appended in the Test Booklet at the end.

DO NOT OPEN THIS BOOKLET UNTIL YOU ARE ASKED TO DO SO


1 www.visionias.in ©Vision IAS

Google it:- https://www.pdfnotes.co


https://www.pdfnotes.co << Download From >> https://
www.pdfnotes.co
1. Consider the following statements: 4. Consider the following factors of migration:
1. China has the largest lithium reserves in 1. Unemployment
the world. 2. Natural Disasters
2. There are no local reserves of lithium in 3. Pleasant Climate
India. 4. Political Turmoil
3. The famous 'lithium triangle' is a 5. Epidemics
lithium-rich region in Eurasia. Which of the factors given above can be
Which of the statements given above is/are termed as push factors for migration?
correct? (a) 2, 3 and 4 only
(a) 1 and 2 only (b) 1, 2, 4 and 5 only
(b) 2 and 3 only (c) 1, 4 and 5 only
(c) 3 only (d) 1, 2, 3 and 5 only
(d) None

5. What is common to the places known as


2. With reference to the various approaches to Ambala, Mhow, Udhampur?
Human Development, consider the following (a) Coal deposits
statements:
(b) Water reservoirs
1. The Welfare approach to human
(c) Military cantonments
development advocates that the level of
(d) Inland ports
income reflects the level of freedom
enjoyed by an individual.
6. Consider the following pairs:
2. The Basic needs approach ignores the
Place Located near
aspect of human choices in human
1. Srinagar : Zanskar Mountain Ranges
development.
2. Kohima : Naga Hills
3. Capability approach is based on building
3. Aizawl : Patkaibum
human capabilities in the areas of health,
Which of the pairs given above are not
education and access to resources.
correctly matched?
Which of the statements given above is/are
(a) 2 and 3 only
correct?
(b) 1 and 3 only
(a) 1, 2 and 3
(c) 1, 2 and 3
(b) 3 only
(d) 1 and 2 only
(c) 2 and 3 only
(d) 1 and 2 only
7. Recently Rome declaration was adopted at

3. Chennai is often called the 'Detroit of India'. the Global Health Summit which was

In this context, Detroit is famous for and has organized by

in common with Chennai which of the (a) World Health Organisation (WHO)

following industries? (b) Organisation for Economic Co-operation

(a) Silk production and Development (OECD)


(b) Automobile (c) European Commission and Italy
(c) Shipbuilding (d) United Nations Development
(d) Diamond processing Programme (UNDP)
2 ©Vision IAS

Google it:- https://www.pdfnotes.co


https://www.pdfnotes.co << Download From >> https://
www.pdfnotes.co
8. Which of the following parameters is/are 11. Which of the following statements is/are
taken into account while calculating the correct with respect to population growth in
Human Poverty Index? India (1921-51)?
1. Household income 1. It is referred to as the period of the
2. Adult literacy rate stationary phase of population growth.
3. Number of small children who are 2. In this period, the birth rate remained
underweight high whereas the mortality rate declined.
Select the correct answer using the code 3. During this period, a negative decadal
given below. population growth rate was recorded.

(a) 1, 2 and 3 Select the correct answer using the code


given below.
(b) 2 only
(a) 2 only
(c) 2 and 3 only
(b) 1 and 3 only
(d) 1 and 3 only
(c) 2 and 3 only
(d) 1, 2 and 3
9. Consider the following statements:
1. A migrant is both an immigrant and an
12. Which of the following factors influence the
emigrant at the same time.
location of Urban Settlements?
2. Migrants who move into a new place are
1. Proximity to trade routes
called emigrants while migrants who 2. Natural Defence
move out of a place are called 3. Raw material and local energy supplies
immigrants. 4. Valuable minerals
Which of the statements given above is/are Select the correct answer using the code
correct? given below.
(a) 1 only (a) 1, 2, 3 and 4
(b) 2 only (b) 2 and 4 only
(c) Both 1 and 2 (c) 1,3 and 4 only
(d) Neither 1 nor 2 (d) 1 and 4 only

10. Consider the following statements: 13. Consider the following statements with
1. Crude Birth Rate (CBR) is defined as the respect to rural settlements in India:
number of births in a year per ten 1. In semi-clustered settlements, the land-
thousand of mid-year population in a owning dominant community tends to
particular region. occupy the central part of the village.
2. Natural Growth of Population is the 2. Hamleted settlements are frequently
population increased by the difference found in the lower Ganga plains.

between birth and death rates plus net 3. Fragmented nature of terrain often
results in dispersed settlements.
migration.
Which of the statements given above are
Which of the statements given above is/are
correct?
correct?
(a) 1 and 2 only
(a) 1 only
(b) 2 and 3 only
(b) 2 only
(c) 1 and 3 only
(c) Both 1 and 2
(d) 1, 2 and 3
(d) Neither 1 nor 2
3 www.visionias.in ©Vision IAS

Google it:- https://www.pdfnotes.co


https://www.pdfnotes.co << Download From >> https://
www.pdfnotes.co
14. Which of the following are part of the 17. Which of the following is the correct
definition of a town as per the Census of sequence of the given countries in the
India? decreasing order of their population density?
1. A minimum population of 5000 (a) England - China - South Africa - Canada
2. Density of the population is at least 600 (b) China - South Africa - Canada - England
persons per sq. km. (c) China - England - Canada - South Africa
3. At least 75 percent of the male working (d) South Africa - China - England - Canada
population engaged in non-agricultural
pursuits
18. Consider the following statements with
Select the correct answer using the code
reference to sweet crude oil and sour crude
given below.
oil:
(a) 1 and 2 only
1. Sweet crude contains a higher amount of
(b) 2 and 3 only
sulfur than sour crude.
(c) 1 and 3 only
2. While Venezuela is a leading producer
(d) 1, 2 and 3
of sour crude, the Appalachian Basin in
North America is a major sweet-crude
15. Consider the following pairs:
location.
Pattern of Features
settlement Which of the statements given above is/are

1. Linear : All roads cut each correct?


pattern other at right angles (a) 1 only
2. Circular : Village is developed (b) 2 only
pattern around lakes or tanks. (c) Both 1 and 2
3. Rectangular : Houses are built along (d) Neither 1 nor 2
pattern roads or railway lines.
Which of the pairs given above is/are 19. Katkari tribal community was recently in the
correctly matched? news for utilising the Pradhan Mantari Van
(a) 1 only Dhan Yojana (PMVDY) for processing and
(b) 2 only marketing Giloy (Tinospora cordifolia) an
(c) 1 and 3 only antipyretic herb. Consider the following
(d) 1, 2 and 3 statements regarding Katkari tribe:
1. Katkaris are predominantly found in
16. What is 'Beed District Formula' recently Odisha.
seen in the news is
2. The tribe is classified as a Particularly
(a) supply chain management model
Vulnerable Tribal Group (PVTG).
agricultural commodities.
Which of the statements given above is/are
(b) a crop insurance model under Pradhan
correct?
Mantri Fasal Bima Yojana (PMFBY).
(a) 1 only
(c) a project management under Pradhan
(b) 2 only
Mantri Gram Sadak Yojana (PMGSY).
(c) Both 1 and 2
(d) a formula related to nutrition
(d) Neither 1 and 2
management in Aspirational Districts.
4 www.visionias.in ©Vision IAS

Google it:- https://www.pdfnotes.co


https://www.pdfnotes.co << Download From >> https://
www.pdfnotes.co
20. Which of the following agrarian reforms 23. With reference to steroids recently seen in
were undertaken by the Government of India the news, consider the following statements:
immediately after independence? 1. They are artificially produced version of
1. Abolition of the zamindari system. hormones normally found in the body.
2. Tenancy abolition and regulation acts. 2. They are used to suppress the immune
3. Regulation of land acquisition
system.
Select the correct answer using the code
Which of the statements given above is/are
given below.
correct?
(a) 1 and 2 only
(a) 1 only
(b) 1, 2 and 3
(b) 2 only
(c) 2 and 3 only
(d) 1 and 3 only (c) Both 1 and 2
(d) Neither 1 and 2
21. With reference to the National Building
Code (NBC), consider the following 24. The rural settlement is fragmented into
statements: several units physically separated from each
1. It covers the detailed guidelines for other bearing a common name. These units
construction, maintenance and fire safety are locally called panna, para, palli, nagla,
of the structures. dhani in various parts of the country. This
2. They are published by National Disaster
segmentation of a large village is often
Management Authority.
motivated by social and ethnic factors.
3. Their guidelines are recommendatory in
The above passage refers to which of the
nature.
following types of rural settlements?
Which of the statements given above is/are
(a) Clustered settlements
correct?
(a) 2 only (b) Semi-clustered settlements
(b) 1 and 3 only (c) Hamleted settlements
(c) 2 and 3 only (d) Dispersed settlements
(d) 1, 2 and 3
25. With reference to rural settlements, consider
22. Consider the following statements with the following statements:
respect to social indicators in India: 1. The design and use of building materials
1. The life expectancy at birth in males is of houses in rural areas vary from one
higher than the females. ecological region to another.
2. Child Sex Ratio in economically
2. The occupation of the population in rural
developed states like Haryana and
settlements is predominantly based on
Punjab is more than the national
secondary and tertiary sector activities.
average.
Which of the statements given above is/are
Which of the statements given above is/are
not correct? correct?
(a) 1 only (a) 1 only
(b) 2 only (b) 2 only
(c) Both 1 and 2 (c) Both 1 and 2
(d) Neither 1 nor 2 (d) Neither 1 nor 2
5 www.visionias.in ©Vision IAS

Google it:- https://www.pdfnotes.co


https://www.pdfnotes.co << Download From >> https://
www.pdfnotes.co
26. This is an important river-based inland 29. Consider the following statements regarding
waterway in Eastern Europe. The river Bauxite:
(waterway) rises in the Black Forest and 1. Bauxite is a non-ferrous metallic mineral
flows eastwards through many countries.
used in the manufacturing of aluminum.
This is the second-longest river in Europe.
2. It is found extensively in the terai region
The river ends in the Black Sea through the
Danube Delta. of the Northern Plains.
Which of the following river waterways is Which of the statements given above is/are
being described in the above passage? correct?
(a) Rhine (a) 1 only
(b) Danube
(b) 2 only
(c) Volga
(c) Both 1 and 2
(d) Thames
(d) Neither 1 nor 2

27. In the context of Growth and Development,


consider the following statements: 30. Whiteflies are invasive insect species, which
1. Growth is a qualitative phenomenon were recently seen in the news. In this
while Development is a quantitative context, consider the following statements:
phenomenon.
1. They are native to Caribbean Islands and
2. Positive Growth always leads to the
Central America and not found in India.
development of a society.
Which of the statements given above is/are 2. They feed only on cotton crop.
correct? Which of the statements given above is/are
(a) 1 only correct?
(b) 2 only (a) 1 only
(c) Both 1 and 2 (b) 2 only
(d) Neither 1 nor 2
(c) Both 1 and 2
(d) Neither 1 nor 2
28. Recently to achieve self-sufficiency in
pulses production Union Government
formulated a strategy for Kharif 2021. 31. Consider the following statements regarding
Consider the following statements with mineral distribution in the Asian continent:
reference to pulses: 1. The continent produces more than half
1. In the last decade the production of
of the world’s tin.
pulses has increased by about 50% in
2. Kalgoorlie and Coolgardie areas in Asia
India.
have the largest deposits of gold.
2. Pulses constitutes the second largest
sown area within Kharif crops. Which of the statements given above is/are
Which of the above statements given above not correct?
is/are correct? (a) 1 only
(a) 1 only (b) 2 only
(b) 2 only
(c) Both 1 and 2
(c) Both 1 and 2
(d) Neither 1 nor 2
(d) Neither 1 nor 2
6 www.visionias.in ©Vision IAS

Google it:- https://www.pdfnotes.co


https://www.pdfnotes.co << Download From >> https://
www.pdfnotes.co
32. Which of the following tribes are desert 35. Recently, the term 'realized equity' of the
inhabitants? Reserve Bank of India was seen in the news.
1. Orang Asli
What do you understand by realized equity?
2. Bushmen
(a) It is a form of contingency risk buffer of
3. Bindibu
the RBI.
4. Kikuyu
Select the correct answer using the code (b) It is interest paid by RBI to banks on
given below. their holdings.
(a) 1 and 4 only (c) It is the amount of surplus that is
(b) 2 and 3 only transferred from RBI to the Government
(c) 1, 2 and 3 only
of India annually.
(d) 2 and 4 only
(d) It is the interest income generated by the

33. Which of the following tribes practice RBI from the loans given to the

transhumance in India? commercial banks.


1. Bakarwals
2. Gaddis 36. Which of the following statements is not
3. Bhotiyas
correct with respect to the features of Jal
Select the correct answer using the code
Kranti Abhiyan?
given below.
(a) 1 only (a) Creation of a ‘Jal Gram’ in every village

(b) 1 and 2 only in all the water-stressed districts of


(c) 2 and 3 only India.
(d) 1, 2 and 3 (b) Construction of Arsenic-free wells in
selected areas of the country.
34. With reference to different streams of
(c) Identification of model command area of
migration in India, consider the following
about 1000 hectares in different parts of
statements:
1. Female migrants dominate the rural-to- the country.
rural Intrastate migration. (d) Water conservation and artificial
2. Male migrants dominate the rural to recharge
urban Interstate migration.
3. Marriage for females and work &
37. In the context of human geography,
employment for males are the main
Kolkhoz, a system of farming based on
causes for their migration.
Which of the statements given above are social ownership, was introduced in:

correct? (a) Cuba


(a) 1 and 3 only (b) China
(b) 2 and 3 only (c) Soviet Union
(c) 1 and 2 only
(d) Egypt
(d) 1, 2 and 3
7 www.visionias.in ©Vision IAS

Google it:- https://www.pdfnotes.co


https://www.pdfnotes.co << Download From >> https://
www.pdfnotes.co
38. With reference to the Human Development 41. Consider the following statements regarding
Index (HDI), consider the following
International Space Station:
statements:
1. It is earth’s microgravity laboratory
1. Education and Health are the only
parameters of the Human Development enabling space research.
Index.
2. It is placed in the Lower Earth Orbit
2. Health as an indicator is measured
(LEO).
through number of Medical Care Centres
per million population of a country. 3. It is a multinational collaborative project
3. The Human Development Index is involving ISRO.
released by the UNDP.
Which of the statements given above is/are
Which of the statements given above is/are
correct? correct?
(a) 1, 2 and 3 (a) 1 and 2 only
(b) 3 only
(b) 2 only
(c) 1 and 2 only
(d) 2 only (c) 1 and 3 only

(d) 1, 2 and 3
39. Consider the following pairs:
Important Oil field Country
1. Safaniya : Iran 42. Consider the following pairs:
2. Upper Zakum : UAE Seaport State
3. Kashagan : Kazakhstan
1. Deendayal Port : Maharashtra
4. Lula Venezuela
Which of the pairs given above is/are 2. Marmagao Port : Karnataka
correctly matched? 3. Paradip Port : Andhra Pradesh
(a) 1 and 2 only
Which of the pairs given above is/are not
(b) 3 and 4 only
(c) 1 only correctly matched?
(d) 2 and 3 only (a) 2 and 3 only

(b) 1 only
40. Which of the following factors act as a
catalyst in better Human Development of a (c) 1, 2 and 3
country? (d) 1 and 3 only
1. Political Stability
2. High Defence expenditure
3. Greater freedom to people 43. Which of the following tribes is not nomadic
4. Equitable distribution of resources in nature?
Select the correct answer using the code
(a) Hausa
given below.
(a) 1, 2, 3 and 4 (b) Bedouin
(b) 1, 3 and 4 only (c) Kirghiz
(c) 2 and 4 only
(d) Chukchi
(d) 1 and 3 only
8 www.visionias.in ©Vision IAS

Google it:- https://www.pdfnotes.co


https://www.pdfnotes.co << Download From >> https://
www.pdfnotes.co
44. Arrange the following neighboring countries 47. Which of the following crops are grown
of India in the decreasing order based on the primarily in the equatorial region?
number of migrants to India from them as 1. Oil Palm
per Indian Census 2011: 2. Natural Rubber
1. Nepal 3. Cocoa
4. Cotton
2. Pakistan
Select the correct answer using the code
3. Bangladesh
given below.
4. Sri Lanka
(a) 2 and 4 only
Select the correct answer using the code
(b) 3 and 4 only
given below. (c) 1, 2 and 3 only
(a) 3 - 2 - 1 - 4 (d) 1, 2, 3 and 4
(b) 2 - 3 - 4 - 1
(c) 1 - 3 - 4 - 2 48. In the context of settlements, a 'double
(d) 1 - 4 - 3 - 2 village' is defined as the:
(a) rural settlement spread on coastal
45. Which of the following statements is/are regions which often requires shifting due
correct regarding mixed farming? to natural disasters.
1. Mixed farming is characterized by low (b) settlement spread on both sides of an
urban area sharing common occupations.
capital expenditure on agricultural
(c) settlement spread on both sides of a river
inputs.
where there is a bridge or a ferry.
2. Fodder crops are an important
(d) rural settlement where people are
component of mixed farming.
engaged in manufacturing and trading,
Select the correct answer using the code apart from agriculture.
given below.
(a) 1 only 49. Who among the following published the
(b) 2 only report called 'Limits to Growth'?
(c) Both 1 and 2 (a) Brundtland Commission
(d) Neither 1 nor 2 (b) Club of Rome
(c) United Nations Environment Programme
46. 'It is largely a rainfed Kharif crop of (d) Intergovernmental Panel on Climate
drylands. But in southern India, it is Change

cultivated during the rabi season as well. It


50. Consider the following pairs:
covers about 3.6 percent of the total cropped
Ramsar Site State
area in the country. Gujarat is the largest
1. Deepor Beel : Assam
producer of this crop accounting for nearly
2. Hokera wetland : Jammu and Kashmir
forty percent of the production.' 3. Rudrasagar Lake : Uttar Pradesh
Which of the following crops is mentioned Which of the pairs given above are correctly
in the passage given above? matched?
(a) Groundnut (a) 1 and 2 only
(b) Mustard (b) 1, 2 and 3
(c) Gram (c) 1 and 3 only
(d) Sunflower (d) 2 and 3 only
9 www.visionias.in ©Vision IAS

Google it:- https://www.pdfnotes.co


https://www.pdfnotes.co << Download From >> https://
www.pdfnotes.co
51. Consider the following statements regarding 54. The Protected Planet Report is published by
the distribution of minerals across various International Union for the Conservation of
continents:
Nature (IUCN) and
1. Australia is the largest producer of
(a) World Wide Fund for Nature (WWF)
bauxite in the world.
2. Africa is the world’s largest producer of (b) Food and Agriculture Organisation
diamonds and gold. (FAO)
3. Antarctica does not have the presence of (c) Climate Action Network
mineral deposits in commercial
(d) United Nations Environment Programme
quantities.
(UNEP)
Which of the statements given above is/are
correct?
(a) 1 and 2 only 55. With reference to the mineral distribution of
(b) 2 only the world, consider the following pairs:
(c) 3 only Mineral-rich region Country
(d) 1, 2 and 3
1. Ruhr : Germany
2. Pittsburgh : The USA
52. Consider the following pairs:
Tribe Region 3. Sheffield : The U.K.
1. Yakuts : Congo Basin Which of the pairs given above is/are
2. Samoyeds : Siberia correctly matched?
3. Tuaregs : Alaska
(a) 1 only
Which of the pairs given above is/are
(b) 1 and 3 only
correctly matched?
(a) 1 and 3 only (c) 2 and 3 only
(b) 2 and 3 only (d) 1, 2 and 3
(c) 2 only
(d) 1, 2 and 3
56. With reference to cropping patterns of
cereals in India, consider the following
53. With reference to the world population,
consider the following statements: statements:

1. The world population crossed the one 1. More than half of the total cropped area
billion mark in the first half of the is occupied by cereals in India.
th
17 century. 2. Area under wheat cultivation is higher
2. At present, Africa has the highest growth
than the area under rice cultivation.
rate of population among all the
Which of the statements given above is/are
continents.
Which of the statements given above is/are correct?
correct? (a) 1 only
(a) 1 only (b) 2 only
(b) 2 only
(c) Both 1 and 2
(c) Both 1 and 2
(d) Neither 1 nor 2
(d) Neither 1 nor 2
10 www.visionias.in ©Vision IAS

Google it:- https://www.pdfnotes.co


https://www.pdfnotes.co << Download From >> https://
www.pdfnotes.co
57. Consider the following statements regarding 60. Which of the following conditions must be
Age-Sex Pyramids: fulfilled for a Super Blood Moon to occur,
which was observed recently?
1. Developing countries generally have
1. Moon must lie completely within the
triangular pyramids with a wide base.
Umbra region of Earth.
2. Bell-shaped pyramid indicates that the 2. Moon and Earth must be as close as
population is almost constant. possible.
3. Countries like Japan have a narrow 3. It must be a full moon night.
pyramid base. Select the correct answer using the code
given below.
Which of the statements given above is/are
(a) 1 and 2 only
correct?
(b) 2 and 3 only
(a) 1 only (c) 1 and 3 only
(b) 1 and 2 only (d) 1, 2 and 3
(c) 2 and 3 only
(d) 1, 2 and 3 61. Consider the following:
1. Natural disasters
2. Availability of regular work
58. Consider the following statements with
3. Poverty
reference to ‘Manganese’ in India: 4. High population pressure on land
1. It is an important raw material for 5. Local conflicts
manufacturing Ferroalloys. Which of the above-mentioned factors can
be classified as the push factors of migration
2. Manganese deposits are mainly
in India?
associated with Dharwar geological rock
(a) 1, 3 and 5 only
system. (b) 1, 3, 4 and 5 only
3. Odisha produces the most manganese in (c) 2 and 4 only
the country. (d) 1, 2, 3, 4 and 5
Which of the statements given above is/are
62. Consider the following statements with
correct?
respect to watershed management:
(a) 1 only
1. It includes conservation, regeneration,
(b) 1 and 2 only and judicious use of all-natural resources
(c) 2 and 3 only within the watershed.
(d) 1, 2 and 3 2. Community participation is one of the
prominent features of watershed
management.
59. In the context of the busiest world trade
Which of the statements given above is/are
routes, the Big Trunk Route is a part of the correct?
(a) South Pacific Ocean (a) 1 only
(b) North Pacific Ocean (b) 2 only
(c) Southern Indian Ocean (c) Both 1 and 2
(d) Neither 1 nor 2
(d) Northern Atlantic Ocean
11 www.visionias.in ©Vision IAS

Google it:- https://www.pdfnotes.co


https://www.pdfnotes.co << Download From >> https://
www.pdfnotes.co
63. Which of the following correctly describes 66. Consider the following statements regarding
the term 'haloclasty'? the distribution of petroleum:
(a) Variations observed in the brightness of 1. Crude petroleum occurs in sedimentary
rocks of the tertiary period.
the sun due to variation of
2. More than half of the world’s proven oil
electromagnetic flux.
reserves are located in North America.
(b) Artificial restoration of coral reefs by an
Which of the statements given above is/are
electrolytic process.
correct?
(c) The gradual reduction in rewards for (a) 1 only
mining cryptocurrency. (b) 2 only
(d) Rock weathering triggered by the growth (c) Both 1 and 2
of salt crystals. (d) Neither 1 nor 2

67. "They are a nomadic tribe who once


64. Which of the following statements correctly
wandered in central highlands of East Africa
explains the term 'physiological density'?
but now mainly confined to the few square
(a) It is a measurement of total population
miles of reserves in Kenya and Tanzania.
per unit area of a country.
The cattle kept by them are the Zebu cattle
(b) It refers to the ratio to total agricultural with humps and long horns. Cattle are kept
population and the total area of a by every family. They drink the blood as
country. well as the milk of their animals but do not
(c) It refers to the ratio of the total kill them for meat."
population and the net cultivable area in The above passage refers to which of the
following tribes?
a country.
(a) Pygmies
(d) It refers to the ratio of the total
(b) Eskimos
agricultural population and the net
(c) Lapps
cultivable area in a country.
(d) Masai

65. Which of the following factors tend to 68. Which of the following is/are the adverse
promote a high density of population in a impacts of rural to urban migration in India?
region? 1. Imbalance in sex ratio in place of origin
1. River valleys and destination of migrants
2. Increased antisocial activities in place of
2. Hill ranges and peaks
destination of migrants
3. Mineral-rich areas
3. Unplanned growth of settlements in
4. Urbanization
place of destination of migrants
Select the correct answer using the code
Select the correct answer using the code
given below. given below.
(a) 1, 2 and 4 only (a) 2 and 3 only
(b) 2 and 4 only (b) 2 only
(c) 1, 3 and 4 only (c) 1, 2 and 3
(d) 1, 2 and 3 only (d) 1 and 3 only

12 www.visionias.in ©Vision IAS

Google it:- https://www.pdfnotes.co


https://www.pdfnotes.co << Download From >> https://
www.pdfnotes.co
69. Consider the following statements regarding 72. Consider the following statements:
'Currency Chest': 1. The phenomenon of 'population ageing'
1. The money present in the currency chest is more common in developing countries
belongs to the respective banks. than developed countries.
2. The security of currency chests is the 2. In developed countries, the proportion of
subject of the bank in which chests are children in the population has declined
situated. due to a reduction in birth rates.
Which of the statements given above is/are
Which of the statements given above is/are
correct?
correct?
(a) 1 only
(a) 1 only
(b) 2 only
(b) 2 only
(c) Both 1 and 2
(c) Both 1 and 2
(d) Neither 1 nor 2
(d) Neither 1 nor 2

70. Which of the following cities lie on the


73. With reference to agriculture and allied
National Waterway (NW) 3 of India?
1. Kochchi activities in the Mediterranean region,
2. Alappuzha consider the following statements:
3. Kollam 1. This region is an important supplier of
4. Kottapuram olives and figs.
5. Thiruvananthapuram 2. Viticulture is a specialty of the region.
Select the correct answer using the code 3. Rice is the principal foodgrain grown in
given below. the region.
(a) 1, 3, 4 and 5 only Which of the statements given above are
(b) 2 and 5 only correct?
(c) 1, 2, 3 and 4 only (a) 1, 2 and 3
(d) 1, 2, 3, 4 and 5 (b) 1 and 3 only
(c) 2 and 3 only
71. Consider the following statements with (d) 1 and 2 only
reference to the occurrence of various
minerals:
74. The HVJ (Hazira-Vijaipur-Jagdishpur)
1. Metallic minerals are obtained from
Natural Gas Pipeline passes through which
veins and lodes.
of the following states of India?
2. Coal mainly occurs as alluvial deposits
1. Gujarat
in the sands of valley floors.
2. Madhya Pradesh
3. Bauxite is formed as a result of an
3. Haryana
accumulation for long periods under
4. Punjab
great heat and pressure.
Which of the statements given above is/are Select the correct answer using the code

correct? given below.


(a) 1 only (a) 1, 2 and 3 only
(b) 2 and 3 only (b) 1, 2, 3 and 4 only
(c) 1 and 3 only (c) 1 and 2 only
(d) 1, 2 and 3 (d) 3 and 4 only

13 www.visionias.in ©Vision IAS

Google it:- https://www.pdfnotes.co


https://www.pdfnotes.co << Download From >> https://
www.pdfnotes.co
75. Consider the following statements: 78. Consider the following statements:
1. As per the world's average sex ratio 1. Tertiary activities are directly involved
there are more females per thousand in the processing of physical raw
materials and the production of tangible
males in the world.
goods.
2. More than half of the countries in the
2. The highest level of decision-makers or
world exhibit a better sex ratio for policymakers perform quaternary
women. activities.
Which of the statements given above is/are 3. Workers of Quinary activities are often
correct? referred to as ‘gold collar’ professions.
(a) 1 only Which of the statements given above is/are
correct?
(b) 2 only
(a) 1 and 2 only
(c) Both 1 and 2
(b) 2 and 3 only
(d) Neither 1 nor 2
(c) 3 only
(d) 2 only
76. Consider the following statements with
regards to pulses: 79. Which of the following states is the largest
1. These are legume crops that increase the producer of coffee in India?
natural fertility of soils through nitrogen (a) Kerala
(b) Tamil Nadu
fixation.
(c) Karnataka
2. Pulses occupy nearly 25 percent of the
(d) Odisha
total cropped area in the country.
Which of the statements given above is/are 80. Recently A-76 iceberg broke away from
correct? Antarctica's Ronne Ice Shelf becoming the
(a) 1 only largest floating iceberg. This iceberg is
(b) 2 only presently floating in which of the following
seas?
(c) Both 1 and 2
(a) Davie Sea
(d) Neither 1 nor 2
(b) Ross Sea
(c) Amundsen Sea
77. Which of the following states/UTs have a (d) Weddell Sea
literacy rate of above 90% as per census
2011? 81. Which of the following sites in India are
1. Kerala classified as UNESCO World heritage sites?
2. Mizoram 1. Elephanta Caves
2. Ganga ghats in Varanasi
3. Lakshadweep Islands
3. Mountain Railways of India
4. Chandigarh
4. Kancheepuram temples
Select the correct answer using the code Select the correct answer using the code
given below. given below.
(a) 1, 2 and 3 only (a) 1, 2 and 4 only
(b) 3 and 4 only (b) 2 and 4 only
(c) 1, 2 and 4 only (c) 1 and 3 only
(d) 1, 2, 3 and 4
(d) 1, 2, 3 and 4
14 www.visionias.in ©Vision IAS

Google it:- https://www.pdfnotes.co


https://www.pdfnotes.co << Download From >> https://
www.pdfnotes.co
82. With reference to the Pradhan Mantri Kisan 85. Nellore, Mysuru, Coimbatore, Alleppey, and
Samman Nidhi (PM-KISAN) scheme, Bhilwara are important locations in India
consider the following statements: associated with:
1. It is 100% centrally funded scheme.
(a) Mica production
2. Money is transferred in farmer’s account
(b) Petroleum refineries
using direct benefit transfer (DBT), in
two equal instalments. (c) Copper deposits
3. Institutional land holders are not eligible (d) Limestone extraction
to get income support under the scheme.
Which of the statements given above is/are 86. Consider the following statements with
correct?
reference to shale gas:
(a) 1 and 3 only
1. Unlike conventional hydrocarbons, shale
(b) 2 only
(c) 2 and 3 only gas is trapped under permeable rocks.
(d) 1, 2 and 3 2. Extraction of shale gas requires a large
quantity of water.
83. Consider the following statements regarding 3. The USA is estimated to have the
footloose industries:
world's largest shale gas reserves.
1. Footloose industries enjoy relatively a
Which of the statements given above is/are
free choice of location.
2. These industries are generally polluting correct?
industries. (a) 1 and 2 only
Which of the statements given above is/are (b) 2 only
correct? (c) 2 and 3 only
(a) 1 only
(d) 1 and 3 only
(b) 2 only
(c) Both 1 and 2
(d) Neither 1 nor 2 87. Consider the following statements with
respect to the religious composition of the
84. According to the census of India, consider population in India:
the following statements regarding the 1. Buddhists account for less than 1% of
working population in India: the total population in India.
1. Male workers outnumber female
2. Muslims form the majority in
workers in all three sectors of the
Lakshadweep.
economy.
2. The work participation rate is always 3. Christians form the second-highest
higher in the areas of higher levels of population group among the minorities.
economic development. Which of the statements given above are
Which of the statements given above is/are correct?
correct?
(a) 1 and 2 only
(a) 1 only
(b) 2 and 3 only
(b) 2 only
(c) Both 1 and 2 (c) 1 and 3 only
(d) Neither 1 nor 2 (d) 1, 2 and 3

15 www.visionias.in ©Vision IAS

Google it:- https://www.pdfnotes.co


https://www.pdfnotes.co << Download From >> https://
www.pdfnotes.co
88. Consider the following statements regarding 91. Consider the following statements with
Demographic Transition Theory: respect to the Coal reserves in India:

1. In the first stage there is an expansion of 1. The Coal occurrences in India are
mainly distributed along the present-day
the population due to high fertility.
river valleys.
2. In the last stage, both fertility and
2. Gondwana formations make up the most
mortality decline considerably.
of total reserves in India.
Which of the statements given above is/are 3. Raniganj is the largest coalfield in the
correct? country.
(a) 1 only Which of the statements given above is/are
(b) 2 only correct?
(c) Both 1 and 2 (a) 1 and 2 only
(b) 2 only
(d) Neither 1 nor 2
(c) 1 and 3 only
(d) None
89. Recently Tianwen-1 was in news, consider
the following statements with reference to it:
92. Consider the following pairs:
1. It is China’s mission to Mars. Name for shifting Region
2. The mission includes a lander and a cultivation
rover. 1. Ladang : Central America
Which of the statements given above is/are 2. Milpa : Malaysia

correct? 3. Jhum : Northeast India


Which of the pairs given above is/are
(a) 1 only
correctly matched?
(b) 2 only
(a) 1, 2 and 3
(c) Both 1 and 2
(b) 2 and 3 only
(d) Neither 1 and 2 (c) 1 only
(d) 3 only
90. Consider the following pairs:
Coal Mines State 93. Which of the following are the Non-

1. Talcher : Odisha Conventional Energy sources?


1. Natural Gas
2. Korba : Jharkhand
2. Waste and Garbage
3. Neyveli : Tamil Nadu
3. Tides and Waves
4. Namphuk : Assam
4. Firewood
Which of the pairs given above are correctly Select the correct answer using the code
matched? given below.
(a) 1 and 4 only (a) 1 and 2 only
(b) 2 and 4 only (b) 2 and 3 only
(c) 1 and 3 only (c) 3 and 4 only

(d) 2 and 3 only (d) 1 and 4 only


16 www.visionias.in ©Vision IAS

Google it:- https://www.pdfnotes.co


https://www.pdfnotes.co << Download From >> https://
www.pdfnotes.co
94. Consider the following activities: 97. Which among the following are the reasons
1. Extraction of Iron from Iron ore for the development of clustered settlements
2. Food processing in India?
3. Transforming cotton into yarn 1. Security reasons
4. Mining of Iron ore 2. Water Scarcity
Which of the activities given above are 3. Rough terrain
considered secondary activities? 4. Fertile alluvial plains
(a) 1, 2 and 3 only Select the correct answer using the code

(b) 1 and 4 only given below.

(c) 2, 3 and 4 only (a) 1, 2 and 4 only

(d) 1, 3 and 4 only (b) 1 and 3 only


(c) 2, 3 and 4 only
(d) 1, 2, 3 and 4
95. Recently Volatiles Investigating Polar
Exploration Rover (VIPER) mission was
98. This belt is rich in ferrous metals and
seen in the news. Consider the following
bauxite. It also contains high-grade iron ore,
statements regarding VIPER mission:
manganese, and limestone. This belt lacks in
1. It is a lunar mission which will land on
coal deposits except for Neyveli lignite. It
the dark side of the moon.
has good deposits of monazite and thorium
2. It is a mission launched by National
in the country.
Aeronautics and Space Administration
Which of the following mineral belts is
(NASA).
being described in the above-given
Which of the statements given above is/are
passage?
correct?
(a) North-Eastern Peninsular belt
(a) 1 only
(b) Himalayan belt
(b) 2 only
(c) North-Western belt
(c) Both 1 and 2 (d) South- Western belt
(d) Neither 1 and 2

99. Arrange the following language families in


96. Arrange the following Iron ore mines in the decreasing order of the number of people
India from West to East: speaking them in India.
1. Ratnagiri 1. Austric
2. Gua 2. Dravidian
3. Durg 3. Indo-European
4. Mayurbhanj 4. Sino-Tibetan
Select the correct answer using the code Select the correct answer using the code
given below. given below.
(a) 1-2-3-4 (a) 2-1-3-4
(b) 1-3-2-4 (b) 3-1-2-4
(c) 1-3-4-2 (c) 3-2-1-4
(d) 4-3-2-1 (d) 1-2-3-4
17 www.visionias.in ©Vision IAS

Google it:- https://www.pdfnotes.co


https://www.pdfnotes.co << Download From >> https://
www.pdfnotes.co
100. Which among the following events
happened the latest?
(a) Establishment of Border Road
Organisation
(b) Construction of Konkan Railway
(c) Setting up of Inland Waterway Authority
of India
(d) Establishment of Oil India Limited

Copyright © by Vision IAS


All rights are reserved. No part of this document may be reproduced, stored in a retrieval system or transmitted
in any form or by any means, electronic, mechanical, photocopying, recording or otherwise, without prior
permission of Vision IAS.

18 www.visionias.in ©Vision IAS

Google it:- https://www.pdfnotes.co


https://www.pdfnotes.co << Download From >> https://
www.pdfnotes.co

VISIONIAS
www.visionias.in
ANSWERS & EXPLANATIONS
GENERAL STUDIES (P) TEST – 3473 (2022)

Q 1.D
• About Lithium
o It is a chemical element with the symbol Li and atomic number 3.
o It is a soft, silvery-white alkali metal. Under standard conditions, it is the lightest metal and the
lightest solid element.
o Like all alkali metals, lithium is highly reactive and flammable and must be stored in mineral oil.
• Lithium is a key component of batteries and other electronics that are becoming increasingly common
around the world – particularly in the case of rechargeable lithium-ion batteries used in electric-vehicle
manufacturing, as well as larger-scale battery storage.
• According to the US Geological Survey (USGS), there are around 80 million tonnes of identified reserves
globally as of 2019. Bolivia is home to the world’s biggest lithium reserves, one-third of the “lithium
triangle” in South America – which also comprises second and third-placed Argentina and Chile. Hence
statements 1 and 3 are not correct.
• Although the US has the world’s fourth-largest lithium reserves, measured at 6.8 million tonnes according
to the US Geological Survey, production activity in the country is minimal.
• Australia may only rank fifth on the list of largest overall reserves with 6.3 million tonnes, but it was by
far the world’s biggest lithium producer in 2019. China is sixth-placed on the list, with overall
lithium reserves estimated at 4.5 million tonnes. In 2019, the country produced 7,500 tonnes of
metal – the third-highest level worldwide.
• India currently imports all its lithium needs. But recent surveys by the Atomic Minerals Directorate
for Exploration and Research (AMD) have shown the presence of lithium resources in Mandya
district, Karnataka. Hence, statement 2 is not correct.

Q 2.C
• Basic Needs Approach: This approach was initially proposed by the International Labour Organisation
(ILO). Six basic needs i.e.: health, education, food, water supply, sanitation, and housing were identified.
The question of human choices is ignored and the emphasis is on the provision of basic needs of
defined sections. Hence statement 2 is correct.
• Capability Approach: This approach is associated with Prof. Amartya Sen. Building human capabilities
in the areas of health, education, and access to resources is the key to increasing human development.
Hence statement 3 is correct.
• Income Approach: This is one of the oldest approaches to human development. Human development is
seen as being linked to income. The idea is that the level of income reflects the level of freedom an
individual enjoys. The higher the level of income, the higher is the level of human development. Hence
statement 1 is not correct.
• Welfare Approach: This approach looks at human beings as beneficiaries or targets of all
development activities. The approach argues for higher government expenditure on education, health,
social secondary, and amenities. People are not participants in development but only passive
recipients. The government is responsible for increasing levels of human development by maximizing
expenditure on welfare.
Q 3.B
• General Motors, Ford Motor Company, and Chrysler Stellantis North America are often referred to
as the "Big Three", being the largest automakers in the United States of America. All three have
their headquarters in the Detroit area.
• Chennai is nicknamed "The Detroit of India", with more than one-third of India's automobile industry
being based in the city. Chennai is also referred to as the ‘Detroit of India’ with the Indian operations of
1 www.visionias.in ©Vision IAS

Google it:- https://www.pdfnotes.co


https://www.pdfnotes.co << Download From >> https://
www.pdfnotes.co
Ford, Hyundai, Renault, and Nissan headquartered in the city and BMW having an assembly plant on the
outskirts. Chennai accounts for 35% of the country’s automobile component industry and 60 percent of
the country’s automotive exports.
• Hence option (b) is the correct answer.

Q 4.B
• People migrate for various reasons like a better economic, social life, etc. There are two sets of factors
that influence migration
• The Push factors that make the place of origin seem less attractive for reasons like
o High unemployment in the region- It pushes people to migrate for better job opportunities
elsewhere
o Natural Disasters- They push people to migrate to places that are safe.
o Political turmoil- It pushes people to migrate as they people may be targeted due to their
political affiliation, religious denomination, etc.
o Epidemics- They push people to migrate to areas where there is less incidence of pandemics.
o Poor living conditions
• The Pull factors make the place of destination seem more attractive than the place where they live.
o Better job opportunities
o Peace and stability
o Pleasant Climate- People tend to move to such places either as tourists or permanently. For
example, the Mediterranean region.
o Better living conditions
o Better services such as education, communication & health services
• Hence option (b) is the correct answer.

Q 5.C
• Some towns and cities specialize in certain functions and they are known for some specific activities,
products, or services. However, each town performs a number of functions. On the basis of dominant or
specialized functions, Indian cities and towns can be broadly classified as follows:
• Industrial towns: Industries constitute the prime motive force of these cities such as Mumbai, Salem,
Coimbatore, Modinagar, Jamshedpur, Hugli, Bhilai, etc.
• Commercial towns: Towns and cities specializing in trade and commerce are kept in this class. Kolkata,
Saharanpur, Satna, etc. are some examples.
• Garrison Cantonment towns: These towns emerged as garrison towns such as Ambala, Jalandhar,
Mhow, Babina, Udhampur, etc.
o Ambala: It is a city and a municipal corporation in the Ambala district in Haryana. It has a large
Indian Army and Indian Air Force presence within its cantonment area. Ambala separates the Ganges
River network from the Indus River network and is surrounded by two rivers – Ghaggar and Tangri –
to the north and to the south.
o Mhow: Mhow, officially Dr. Ambedkar Nagar, is a cantonment in the Indore district in the Madhya
Pradesh state of India.
o Babina: Babina is a cantonment town in Jhansi district in the state of Uttar Pradesh.
o Udhampur: Udhampur is a city and a municipal committee in the Udhampur district in the Indian
union territory of Jammu and Kashmir. Named after Raja Udham Singh, it serves as the district
capital and the Northern Command headquarters of the Indian Army.
o Hence option (c) is the correct answer.
• Administrative towns and cities: Towns supporting administrative headquarters of higher order are
administrative towns, such as Chandigarh, New Delhi, Bhopal, Shillong, Guwahati, Imphal, Srinagar,
Gandhinagar, Jaipur Chennai, etc.

Q 6.B
• The Kashmir Valley is situated at an average elevation of 1,000 m, is surrounded by the Pir Panjal Range
and the Great Himalaya Range. Within the valley lies Srinagar, the state's largest city and its summer
capital. Hence, pair 1 is not correctly matched.
• Kohima is the capital city of the northeastern state of Nagaland. The town lies in the Naga Hills, 30 miles
(48 km) southeast of the railroad at Dimapur. Hence, pair 2nd is correctly matched.
• Aizawl is the capital city of the northeastern state of Mizoram. It is located in the Mizo hills (formerly
Lushai Hills). Hence, pair 3rd is not correctly matched.

2 www.visionias.in ©Vision IAS

Google it:- https://www.pdfnotes.co


https://www.pdfnotes.co << Download From >> https://
www.pdfnotes.co
Q 7.C
• Recent Context: On 21 May 2021 in Rome, the European Commission and Italy, as chair of the
G20, co-hosted the Global Health Summit 2021. The Summit was an opportunity for G20 and invited
leaders, heads of international and regional organisations, and representatives of global health bodies, to
share lessons learned from the COVID-19 pandemic. The leaders adopted a ‘Rome Declaration’,
committing to common principles to overcome COVID-19 and to prevent and prepare for future
pandemics. Hence, option (c) is the correct answer.
• Leaders of the G20 committed to a series of actions to accelerate the end of the COVID-19 crisis
everywhere and better prepare for future pandemics.
• Key features of Rome Declaration:
o Support and enhance the existing multilateral health architecture for better preparedness, prevention,
detection and response.
o Appropriate and sustainable funding for collaborative global response efforts, especially the Access to
COVID-19 Tools (ACT) Accelerator.
o Foster all-of-society and health-in-all policies approaches.
o Invest in the worldwide health and care workforce.
o Promote the multilateral trading system and open, resilient, diversified, secure, efficient and reliable
global supply chains related to health emergencies, etc.

Q 8.C
• The Human Poverty Index (HPI) was an indication of the poverty of a community in a country, developed
by the United Nations to complement the Human Development Index (HDI) and was first reported as part
of the Human Deprivation Report in 1997. In 2010, it was supplanted by the UN's Multidimensional
Poverty Index.
• The Human Poverty Index measures shortfalls in Human Development. It is a non-income measure.
Hence option 1 is not correct
• Parameters that are taken into account while calculating HPI are:
o The probability of not surviving till the age of 40
o The adult illiteracy rate
o The number of people who do not have access to clean water
o The number of small children who are underweight. Hence option 2 and option 3 are correct.
• Since 1990, the United Nations Development Programme (UNDP) has been publishing the Human
Development Report every year. This report provides a rank-wise list of all member countries according
to the level of human development. The Human Development Index and the Human Poverty index are
two important indices to measure human development used by the UNDP.

Q 9.A
• Human migration involves the movement of people from one place to another with intentions of settling,
permanently or temporarily, at a new location.
• Migration may be permanent, temporary, or seasonal. It may take place from rural-to-rural areas, rural-to-
urban areas, urban-to-urban areas, and urban-to-rural areas. Migration may also be voluntary or forced.
• Migrants who move into a new place are called Immigrants. Migrants who move out of a place are
called Emigrants. Hence statement 2 is not correct.
• If a person is moving from place A to place B then from the lens of place A he is called an emigrant
and from the lens of place B he is called an immigrant. Hence statement 1 is correct.

Q 10.D
• There are three components of population change – births, deaths, and migration.
• Crude Birth Rate (CBR) is expressed as number of "live" births in a year per thousand of mid-year
population. It is calculated as:
o CBR = (Live births during the year / Mid-year population of the area) * 1000
• Hence statement 1 is not correct.
• Crude Death Rate (CDR) is expressed as the number of deaths in a year per thousand of mid-year
population. It is calculated as:
o CDR = (Number of Deaths / Mid-year population of the area) * 1000
• Natural Growth of Population is the population increased by the difference between births and
deaths in a particular region between two points of time. It does not include change due to
migration. Hence statement 2 is not correct.
• The actual growth of population = Birth - Death + In migration - Out Migration
3 www.visionias.in ©Vision IAS

Google it:- https://www.pdfnotes.co


https://www.pdfnotes.co << Download From >> https://
www.pdfnotes.co
Q 11.A
• The growth rate of population in India over the last century has been caused by the annual birth rate and
death rate and rate of migration and thereby show different trends. There are four distinct phases of
growth identified within this period:
• Phase 1: The period from 1901-1921 is referred to as a period of the stagnant or stationary phase of
growth of India’s population, since in this period growth rate was very low, even recording a negative
growth rate during 1911-1921. In this period both the birth rate and death rate were high keeping the rate
of increase low.
• Phase 2: The decades 1921-1951 are referred to as the period of steady population growth. An overall
improvement in health and sanitation throughout the country brought down the mortality rate. At the
same time, better transport and communication system improved distribution system. The crude birth
rate remained high in this period leading to a higher growth rate than the previous phase. This is
impressive in the backdrop of the Great Economic Depression, the 1920s, and World War II. Hence
option (a) is the correct answer.
• Phase 3: The decades 1951-1981 are referred to as the period of population explosion in India, which
was caused by a rapid fall in the mortality rate but a high fertility rate of population in the country.
Besides, increased international migration bringing in Tibetans, Bangladeshis, Nepalese and even people
from Pakistan contributed to the high growth rate.
• Phase 4: In the post-1981 till present, the growth rate of the country’s population though remained high,
has started slowing down gradually.

Q 12.A
• The location of urban centres is examined with reference to their function. For example, the sitting
requirements of a holiday resort are quite\ different from that of an industrial town, a military centre or a
seaport. Strategic towns require sites offering natural defence; mining towns require the presence of
economically valuable minerals; industrial towns generally need local energy supplies or raw
materials. Hence statements 1, 2 and 3 are correct.
• Locations of the earliest urban settlements were based on the availability of water, building materials and
fertile land.
• Some of these functional factors do not necessarily require the urban centre to have any fundamental
relationship with its neighbouring rural areas.
• Apart from the site of an urban settlement, the situation plays an important role in the expansion of
towns. The urban centres which are located close to an important trade route have experienced rapid
development. Hence statement 2 is correct.

Q 13.D
• There are various factors and conditions responsible for having different types of rural settlements in
India. These include:
o physical features – nature of terrain, altitude, climate, and availability of water;
o cultural and ethnic factors – social structure, caste, and religion;
o security factors – defence against thefts and robberies.
• Rural settlements in India can broadly be put into four types: Clustered; Semi-clustered or fragmented;
Hamleted, and Dispersed.
• Semi-Clustered Settlements: These may result from the tendency of clustering in a restricted area of
dispersed settlement. Generally, the land-owning and dominant community occupy the central part of
the main village, whereas people of lower strata of society and menial workers settle on the outer flanks of
the village. Such settlements are widespread in the Gujarat plain and some parts of Rajasthan. Hence
statement 1 is correct.
• Hamleted Settlements: Sometimes settlement is fragmented into several units physically separated from
each other bearing a common name. These units are locally called Panna, para, Palli, nagla, Dhani, etc.
in various parts of the country. This segmentation of a large village is often motivated by social and ethnic
factors. Such villages are more frequently found in the middle and lower Ganga plain, Chhattisgarh,
and lower valleys of the Himalayas. Hence statement 2 is correct.
• Dispersed Settlements: Dispersed or isolated settlement pattern in India appears in the form of isolated
huts or hamlets of few huts in remote jungles, or on small hills with farms or pasture on the slopes.
Extreme dispersion of settlement is often caused by the extremely fragmented nature of the terrain and
land resource base of habitable areas. Many areas of Meghalaya, Uttaranchal, Himachal Pradesh, and
Kerala have this type of settlement. Hence statement 3 is correct.

4 www.visionias.in ©Vision IAS

Google it:- https://www.pdfnotes.co


https://www.pdfnotes.co << Download From >> https://
www.pdfnotes.co
Q 14.C
• The definition of urban areas varies from one country to another. Some of the common basis of
classification is the size of the population, occupational structure, and administrative structure.
• For the Census of India 2011, the definition of the urban area is as follows;
o All places with a municipality, corporation, cantonment board or notified town area committee, etc.
o All other places which satisfied the following criteria:
§ A minimum population of 5,000;
§ At least 75 percent of the male main working population engaged in non-agricultural pursuits;
and
§ A density of population of at least 400 persons per sq. km.
o Hence option (c) is the correct answer.
• The first category of urban units is known as Statutory Towns. These towns are notified under the law by
the concerned State/UT Government and have local bodies like municipal corporations, municipalities,
municipal committees, etc., irrespective of their demographic characteristics.

Q 15.B
• Rural settlements may be classified on the basis of a number of criteria, One of them is on the basis of
shapes of settlements.
o Linear pattern: In such settlements houses are located along a road, railway line, river, canal edge of
a valley, or along a levee.
o Rectangular pattern: Such patterns of rural settlements are found in plain areas or wide
intermontane valleys. The roads are rectangular and cut each other at right angles. Hence pairs 1
and 3 are not correctly matched.
o Star-like pattern: Where several roads converge, star-shaped settlements develop by the houses
built along the roads.
o Circular pattern: Circular villages develop around lakes, tanks and sometimes the village is planned
in such a way that the central part remains open and is used for keeping the animals to protect
them from wild animals. Hence pair 2 is correctly matched.
o T-shaped, Y-shaped, Cross-shaped, or cruciform settlements: T-shaped settlements develop at tri-
junctions of the roads while Y-shaped settlements emerge as the places where two roads converge on
the third one and houses are built along these roads. Cruciform settlements develop on the crossroads
and houses extend in all four directions.

Q 16.B
• Recent context: Under the Pradhan Mantri Fasal Bima Yojana (PMFBM) state governments have to pay
the premium for the crop insurance. This year the Indian Meteorological Department (IMD) has predicted
a normal monsoon season. Thus already financially stressed states (due to pandemic) demand that ‘Beed
District Formula’ also called the ‘80-110 plan’ should be adopted to ensure that premium above a
threshold is refunded to the state governments.
• What is Beed district Formula (80-110 plan): It was implemented in Beed district (Maharashtra) and
also in Madhya Pradesh as special cases last year.
o Under the 80-110 plan, the insurer’s potential losses are limited.
o The firm will not have to pay any claims above 110% of the gross premium.
o The state government has to bear the cost of any claims above 110% of the premium collected to
insulate the insurer from losses.
o The premium surplus (gross premium minus claims) exceeding 20% of gross premium is refunded by
insurer to the state government.
o For instance, under the ’80-110 Plan’, in case the claims reach 60% of premium collected, the
insurance company will have to refund 20% to the state government and if the claims are 70%, the
refund to state will be 10%. In case of claims above 80%, the state will not get any refund.
o Under PMFBY, premium to be paid by farmers is fixed at 1.5% of the sum insured for rabi
crops and 2% for kharif crops, while it is 5% for cash crops. The balance premium is split
equally between the Centre and states.
• Hence, option (b) is the correct answer.

Q 17.A
• The ratio between the numbers of people to the size of the land is the density of population. It is usually
measured in persons per sq km.
• Density of Population = Population / Area
5 www.visionias.in ©Vision IAS

Google it:- https://www.pdfnotes.co


https://www.pdfnotes.co << Download From >> https://
www.pdfnotes.co
• For example, area of Region X is 100 sq km and the population is 1,50,000 persons. The density of
population is calculated as:
o Density = 1,50,000 / 100 = 1,500 person/sq km
• The densely populated parts of the world with more than 200 persons on every sq km are the North-
Eastern part of U.S.A., North-Western part of Europe, South, South-East and East Asia. For
example, England- 432, China - 153 persons per sq km.
• Other areas like those near the North and South Poles, the hot and the cold deserts and high rainfall zones
near the Equator have a very low density of population. These are the sparsely populated regions of the
world with less than 01 person per sq km. For example, Canada- 4 persons per square km.
• In between these two types are the areas of medium density. There are 11 to 50 persons per sq km in these
areas. Norway, Sweden in Europe, South Africa (48 persons per sq km) are some examples.
• Hence option (a) is the correct answer.

Q 18.B
• Classification of crude oil
o Crude oil may be referred to as sweet crude if it contains relatively little sulfur (0.5%) or sour crude
if it contains substantial amounts of sulfur.
o Iraq is one of the leading producers of sweet crude.
o Major locations where sweet crude is found include the Appalachian Basin in Eastern North
America, Western Texas, the Bakken Formation of North Dakota and Saskatchewan, the North
Sea of Europe, North Africa, Australia, and the Far East including Indonesia.
o Sour crude, on the other hand, has a high level of impurities in it, namely sulfur, which must first
be removed before being processed into gas and other petroleum-based products.
o Venezuela is a leading producer of sour crude oil. Sour crude is more common in the Gulf of
Mexico, Mexico, South America, and Canada.
o Crude produced by OPEC Member Nations also tends to be relatively sour, with an average sulfur
content of 1.77%.
o Hence option (b) is the correct answer.

Q 19.B
• Recent Context: The Katkari tribal community was recently in news for utilising the Pradhan Mantari
Van Dhan Yojana (PMVDY) for processing and marketing Giloy (Tinospora cordifolia) an antipyretic
herb. Giloy is used as a medicine to boost immunity and to treat various fevers and other conditions.
6 www.visionias.in ©Vision IAS

Google it:- https://www.pdfnotes.co


https://www.pdfnotes.co << Download From >> https://
www.pdfnotes.co
• The Katkari tribes are located primarily in Raigad and in parts of Palghar, Ratnagiri and Thane
districts of Maharashtra as well and in some places of Gujarat. Katkaris were historically forest
dwellers. The name Katkari is derived from a forest based activity – the making and barter or sale of
Katechu (kath) from the khair tree (Acacia Katechu). It is produced by boiling wood from the Khair tree
and evaporating the resulting brew. Hence, statement 1 is not correct.
• Presently, the Katkari is one of the 75 Particularly Vulnerable Tribal Groups, as per the
classification by Ministry of Home Affairs. The Government of India came up with this classification to
introduce targeted interventions noting that some tribal groups had the least development indices as
compared to other tribal groups. Hence, statement 2 is correct.
• There are certain tribal communities who use a pre-agricultural level of technology, face stagnant or
declining population growth, and are equipped with only an extremely low level of literacy and a
subsistence level of economy. 75 such groups of tribals in 18 States and one Union Territory have been
identified and categorized as Particularly Vulnerable Tribal Groups (PVTGs).
Q 20.A
• From the 1950s to the 1970s, a series of land reform laws were passed – at the national level as well as
in the states – that were intended to bring reform in the agrarian structure, especially in the
landholding system and the distribution of land.
• The first important legislation was the abolition of the zamindari system, which removed the layer
of intermediaries who stood between the cultivators and the state. Of all the land reform laws that
were passed, this was probably the most effective, for in most areas it succeeded in taking away the
superior rights of the zamindars over the land and weakening their economic and political power.
Hence statement 1 is correct.
• Among the other major land reform laws that were introduced were the tenancy abolition and regulation
acts. They attempted either to outlaw tenancy altogether or to regulate rents to give some security
to the tenants. Hence statement 2 is correct.
• The third major category of land reform laws was the Land Ceiling Acts. These laws imposed an
upper limit on the amount of land that can be owned by a particular family. The ceiling varies from
region to region, depending on the kind of land, its productivity, and other such factors. Very productive
land has a low ceiling while unproductive dry land has a higher ceiling limit. According to these acts, the
state is supposed to identify and take possession of surplus land (above the ceiling limit) held by each
household and redistribute it to landless families and households in other specified categories, such as SCs
and STs.
• The Right to Fair Compensation and Transparency in Land Acquisition, Rehabilitation and
Resettlement Act, 2013 (also Land Acquisition Act, 2013) was passed to regulate the land
acquisition process and lay down the procedure and rules for granting compensation,
rehabilitation, and resettlement to the affected persons in India. The Act has provisions to provide fair
compensation to those whose land is taken away, brings transparency to the process of acquisition of land
to set up factories or buildings, infrastructural projects, and assures rehabilitation of those affected. Hence
statement 3 is not correct.

Q 21.B
• Recent Context: Occurrence of fire in public buildings of India is a common phenomenon. This results
into death of many citizens and injuring many more. In the last year there have been deadly fires in
hospitals, residential and commercial buildings, including those treating COVID-19 patients. According to
the National Crime Records Bureau (NCRB) 330 people died in commercial building fires in 2019, while
fatalities for residential or dwelling buildings were much higher at 6,329.
• National Building Code of India covers the detailed guidelines for construction, maintenance and
fire safety of the structures. Part four of the document deals with fire and life safety. This document
provides specifications and guidelines for design and material that reduce the threat of destructive fire.
Under this code all the existing and new buildings are classified by their nature. Hence, statement 1 is
correct.
• The National Building Code of India is published by Bureau of Indian Standards (BIS). This code
gets the state government to make their bylaws in a manner which ensures safety of the structures. Hence,
statement 2 is not correct.
• The National Building Code is a recommendatory document. States are not mandated to implement
it as it is. This document provides guidelines to state governments to incorporate those guidelines into
their local laws. Incorporation of these guidelines into the local bylaws makes them mandatory for the
purpose of the construction of structures. Hence by itself the guidelines under the National Building Code
are not mandatory and are only recommendatory. Hence, statement 3 is correct.
7 www.visionias.in ©Vision IAS

Google it:- https://www.pdfnotes.co


https://www.pdfnotes.co << Download From >> https://
www.pdfnotes.co
Q 22.C
• India has done reasonably well in some of the health indicators like the decline in the death rate from 25.1
per thousand in 1951 to 6.5 per thousand in 2015 and infant mortality from 148 per thousand to 37 during
the same period. Similarly, it also succeeded in increasing life expectancy at birth from 37.1 years to
66.9 years for males and 36.2 to 70 years for females from 1951 to 2015. Though these are great
achievements, a lot needs to be done. Similarly, it has also done reasonably well in bringing down the
birth rate from 40.8 to 20.8 during the same years, but it still is much higher than many developed
countries. Hence statement 1 is not correct.
• The situation is more alarming when seen in the context of gender-specific and rural and urban health
indicators. India has recorded a declining female sex ratio. The findings of the 2011 Census of India are
very disturbing particularly in the case of the child sex ratio (919) in the age group of 0-6 years. The
other significant features of the report are, with the exception of Kerala, the child sex ratio has declined in
all the states and it is the most alarming in the developed state of Haryana (834) and Punjab (846)
where it is below 850 female children per thousand male children. Hence statement 2 is not correct.

Q 23.C
• Recent context: Steroids are widely being used to treat severe Covid-19 infection.
• Steroids are artificially produced version of hormones normally produced in our body. Hence
statement 1 is correct.
• The two main types of steroids are: corticosteroids and anabolic-androgenic steroids (or anabolics).
o Corticosteroids are steroid hormones that are either produced by the body or are man-made.
§ Systemic corticosteroids refer to corticosteroids that are given orally or by injection and
distribute throughout the body. It does not include corticosteroids used in the eyes, ears, or
nose, on the skin or that are inhaled, although small amounts of these corticosteroids can be
absorbed into the body. Naturally occurring corticosteroids, hydrocortisone (Cortef) and
cortisone, are produced by the outer portion of the adrenal gland known as the cortex (hence
the name, corticosteroid).
§ Corticosteroids are classified as either: glucocorticoids (anti-inflammatory) which suppress
inflammation and immunity and assist in the breakdown of fats, carbohydrates, and proteins, or
as mineralocorticoids (salt retaining) that regulate the balance of salt and water in the body.
In patients with COVID-19, steroids (such as Dexamethasone) can be used to reduce the damage
induced by the cytokine storm. Hence, statement 2 is correct.
§ Synthetic corticosteroids mimic the actions of naturally occurring corticosteroids and may be
used to replace corticosteroids in people with adrenal glands that are unable to produce adequate
amounts of corticosteroids, however, they more often are used in higher-than-replacement doses
to treat diseases of immunity, inflammation or salt and water balance. Examples of synthetic
corticosteroids include: bethamethasone, (Celestone)prednisone (Prednisone Intensol), etc.
o Anabolic steroids are synthetic (man-made) versions of testosterone. Testosterone is the main sex
hormone in men. It is needed to develop and maintain male sex characteristics, such as facial hair,
deep voice, and muscle growth. Women do have some testosterone in their bodies, but in much
smaller amounts. Health care providers use anabolic steroids to treat some hormone problems in men,
delayed puberty, and muscle loss from some diseases. But some people misuse anabolic steroids
especially bodybuilders and athletes use anabolic steroids to build muscles and improve athletic
performance.
Q 24.C
• There are various factors and conditions responsible for having different types of rural settlements in
India.
o Physical features – nature of terrain, altitude, climate and availability of water
o Cultural and ethnic factors – social structure, caste and religion
o Security factors – defence against thefts and robberies.
• Rural settlements in India can broadly be put into four types:
o Clustered Settlements:
§ The clustered rural settlement is a compact or closely built up area of houses.
§ The general living area is distinct and separated from the surrounding farms, barns and pastures.
§ The closely built-up area and its intervening streets present some recognisable pattern or
geometric shape, such as rectangular, radial, linear, etc.
§ Such settlements are generally found in fertile alluvial plains and in the northeastern states.
> Sometimes, people live in the compact village for security or defence reasons, such as in the
Bundelkhand region of central India and in Nagaland.
8 www.visionias.in ©Vision IAS

Google it:- https://www.pdfnotes.co


https://www.pdfnotes.co << Download From >> https://
www.pdfnotes.co
> In Rajasthan, scarcity of water has necessitated compact settlement for maximum utilisation
of available water resources.
o Semi-Clustered Settlements:
§ Semi-clustered or fragmented settlements may result from the tendency of clustering in a
restricted area of dispersed settlement.
§ More often such a pattern may also result from segregation or fragmentation of a large compact
village. In this case, one or more sections of the village society choose or is forced to live a little
away from the main cluster or village. In such cases, generally, the land-owning and dominant
community occupies the central part of the main village, whereas people of lower strata of society
and menial workers settle on the outer flanks of the village.
§ Such settlements are widespread in the Gujarat plain and some parts of Rajasthan.
o Hamleted Settlements:
§ Sometimes a settlement is fragmented into several units physically separated from each other
bearing a common name.
§ These units are locally called panna, para, palli, nagla, dhani, etc. in various parts of the
country.
§ This segmentation of a large village is often motivated by social and ethnic factors.
§ Such villages are more frequently found in the middle and lower Ganga plain, Chhattisgarh and
lower valleys of the Himalayas.
§ Hence option (c) is the correct answer.
o Dispersed Settlements:
§ The dispersed or isolated settlement pattern in India appears in the form of isolated huts or
hamlets of few huts in remote jungles, or on small hills with farms or pasture on the slopes.
§ Extreme dispersion of settlement is often caused by the extremely fragmented nature of the terrain
and land resource base of habitable areas.
§ Many areas of Meghalaya, Uttaranchal, Himachal Pradesh and Kerala have this type of
settlement.

Q 25.A
• Rural settlements are most closely and directly related to land. They are dominated by primary activities
such as agriculture, animal husbandry, fishing, etc. The settlement's size is relatively small. Hence
statement 2 is not correct.
• Upland which is not prone to flooding is chosen for rural settlements to prevent damage to houses and loss
of life. Thus, in low-lying river basins, people chose to settle on terraces and levees which are “dry
points”. In tropical countries, people build their houses on stilts near marshy lands to protect themselves
from floods, insects, and animal pests.
• A levee is a natural or artificial wall that blocks water. Levees may be used to increase available land for
habitation or divert a body of water so the fertile soil of a river or sea bed may be used for agriculture.
They prevent rivers from flooding cities in a storm surge.
• The availability of building materials- wood, stone near settlements is another advantage. Early villages
were built in forest clearings where wood was plentiful.
• The design and use of building materials of houses in rural areas vary from one ecological region to
another. The houses made up of mud, wood and thatch, remain susceptible to damage during heavy rains
and floods, and require proper maintenance every year. We often observe that houses in hilly areas have
roofs that are tilted so that the melted snow drains easily from the roof to the ground. Hence statement 1
is correct.

Q 26.B
• Inland Waterways:
o The Rhine Waterways: The Rhine flows through Germany and the Netherlands. It is navigable for
700 km from Rotterdam, at its mouth in the Netherlands to Basel in Switzerland. Ocean-going vessels
can reach up to Cologne. The Ruhr river joins the Rhine from the east. It flows through a rich
coalfield and the whole basin has become a prosperous manufacturing area. Dusseldorf is the Rhine
port for this region. Huge tonnage moves along the stretch south of the Ruhr. This waterway is the
world’s most heavily used. Each year more than 20,000 ocean-going ships and 2,00,000 inland
vessels exchange their cargoes. It connects the industrial areas of Switzerland, Germany, France,
Belgium, and the Netherlands with the North Atlantic Sea Route.
o The Danube Waterway: This important inland waterway serves Eastern Europe. River Danube rises
in the Black Forest and flows eastwards through many countries. It is navigable up to Taurna Severin.
9 www.visionias.in ©Vision IAS

Google it:- https://www.pdfnotes.co


https://www.pdfnotes.co << Download From >> https://
www.pdfnotes.co
The chief export items are wheat, maize, timber, and machinery. The Danube (Latin name Hister) is
the second-longest river in Europe (the Volga is the longest). It is the longest river in the European
Union. The Danube flows through or makes part of the border of 10 countries: Germany, Austria,
Slovakia, Hungary, Croatia, Serbia, Romania, Bulgaria, Moldova, and Ukraine. The river ends in the
Black Sea through the Danube Delta. Hence option (b) is the correct answer.
o The Volga Waterway: Russia has a large number of developed waterways, of which the Volga is one
of the most important. It provides a navigable waterway of 11,200 km and drains into the Caspian
Sea. The Volga-Moscow Canal connects it with the Moscow region and the Volga-Don Canal with
the Black Sea.
o Thames: The River Thames, known alternatively in parts as the River Isis, is a river that flows
through southern England including London. At 215 miles, it is the longest river entirely in England
and the second-longest in the United Kingdom, after the River Severn.

Q 27.D
• Both growth and development refer to changes over a period of time. The difference is that growth is
quantitative and value-neutral. It may have a positive or a negative sign. This means that the change
may be either positive (showing an increase) or negative (indicating a decrease). Hence statement 1 is
not correct
• Development means a qualitative change that is always value positive. This means that development
cannot take place unless there is an increment or addition to the existing conditions. Development occurs
when positive growth takes place. Yet, positive growth does not always lead to development.
Development occurs when there is a positive change in quality. Hence statement 2 is not correct.

Q 28.B
• Recent Context: Recently with an aim to attain self-sufficiency in the production of pulses, the Ministry
of agriculture and farmers welfare has formulated a special kharif strategy for implementation in the in
ensuing kharif season 2021. Through consultations with the state governments, a detailed plan for both
area expansion and productivity enhancement for Tur, Moong and Urad has been formulated. Under the
strategy, utilising all the high yielding varieties (HYVs) of seeds that are available either with the
Central Seed Agencies or in the States will be distributed free of cost to increase area through
intercropping and sole crop.
• From 18.09 million tonnes in in 2010-2011 to 24.42 million tonnes there has been an increase of around
that 25% in the production of pulses in India. Hence, statement 1 is not correct.
• Sowing area coverage of the Kharif crops has increase by 13.92% as on 31.07.2020, as compared to the
last year. Rice accounts for the highest shown area (266.60 Lakh ha area) where is pulses account for
Second highest area followed by rice which is 111.91 lakh ha. Hence, statement 2 is correct.
• From 2016-2017 onwards, under the national Food security Mission, 644 districts have been included in
the pulse programme. Total number of districts in India at present is 718. Hence though the programme
includes most of the districts and major portion of the country but still it does not cover all the districts of
the country.

Q 29.A
• Bauxite is an ore that is used in the production of aluminium. Hence statement 1 is correct. India is
poorly endowed with non-ferrous metallic minerals except bauxite.
• Bauxite is primarily comprised of aluminum oxide compounds (alumina), silica, iron oxides and
titanium dioxide. Approximately 70 percent of the world’s bauxite production is refined through
the Bayer chemical process into alumina. Alumina is then refined into pure aluminum metal through
the Hall–Héroult electrolytic process.
• Bauxite is found mainly in tertiary deposits and is associated with laterite rocks occurring
extensively either on the plateau or hill ranges of peninsular India and also in the coastal tracts of
the country. Hence statement 2 is not correct.
• Odisha is the largest producer of bauxite. Kalahandi and Sambalpur are the leading producers.
Bolangir and Koraput have also increased their production in the recent years.
• The patlands of Lohardaga in Jharkhand have rich deposits. Gujarat, Chhattisgarh, Madhya
Pradesh and Maharashtra are other major producers.
• Tamil Nadu, Karnataka and Goa are minor producers of bauxite

10 www.visionias.in ©Vision IAS

Google it:- https://www.pdfnotes.co


https://www.pdfnotes.co << Download From >> https://
www.pdfnotes.co
Q 30.D
• Recent Context: The invasive Rugose Spiraling Whitefly (RSW) seems to be showing its presence in the
State with its incidence being reported in different districts. According to information, a high incidence
(more than 30 spirals for leaflet) is reported in East and West Godavari and Nellore districts of Andhra
Pradesh.
• About Whiteflies:
o They are native to the Caribbean islands or Central America or both. In India they are found in
all states and Union Territories except Jammu and Kashmir. There are now eight invasive
whiteflies species found in India. Hence statement 1 is not correct
o They are small white-coloured invasive insects affecting cotton, vegetable crops and other crops
in tropical and sub-tropical regions (also in India).They spread due to their polyphagous nature
(ability to feed on various kinds of food). Invasive whiteflies were also found to expand their host
range on valuable plants species, especially coconut, banana, mango, sapota, guava, cashew, oil palm,
and ornamental plants. Hence, statement 2 is not correct.
o The whiteflies are difficult to control by using synthetic insecticides, and hence currently naturally
occurring insect predators, parasitoids (they are a diverse group of insects that provide biological
control of pests) and entomopathogenic fungi (fungi that can kill insets) are being used. The
naturally found solutions are both environmentally friendly and economically feasible.
• Invasive alien species (IAS): According to IUCN (International union of Conservation of Nature), IAS
are species that are introduced, accidentally or intentionally, outside of their natural geographic range and
that become problematic. All aliens (species) are not invasive species, but all invasives are aliens.
• National Biodiversity Authority (NBA) identifies 169 species as invasive in India. In India, there are
no specific policies to address the issue of IAS, though it is part of several existing biodiversity legislation
and regulations. India has adopted Aichi Target 9 as its National Biodiversity Target 4 – i.e., to
identify invasive alien species and their pathways of introduction, and to develop strategies to
manage IAS.

Q 31.B
• The continent of Asia produces more than half of the world’s tin. Hence statement 1 is correct.
• China, Malaysia, and Indonesia are among the world’s leading tin producers. China also leads in the
production of lead, antimony, and tungsten. Asia also has deposits of manganese, bauxite, nickel, zinc,
and copper.
• Australia is the largest producer of bauxite in the world. It is a leading producer of gold, diamond, iron
ore, tin, and nickel. It is also rich in copper, lead, zinc, and manganese. Kalgoorlie and Coolgardie areas
of western Australia have the largest deposits of gold. Hence statement 2 is not correct.
Q 32.B
• The Orang Asli:
o Orang Asli is a collective term (which means original or first peoples in Malay) for some 18 ethnic
groups of less than 150,000 in total who are widely regarded as comprising
peninsular Malaysia’s original inhabitants (in the sense that they pre-date the arrival of Malays).
o They are generally divided into three distinct groupings: the Negrito, Senoi and Proto-Malay.
o In spite of differences in languages and physical appearances, all Orang Asli tribes share one thing in
common, which is that they are the descendants of the earliest known inhabitants who occupied the
Malay Peninsula.
o They lived in West Malaysia long before the arrival of the other races, that is, the Malays, the
Chinese, and the Indians.
o They are not a desert inhabitant.
• The Bushmen:
o They are nomadic hunters and food gatherers, growing no crops and domesticating no animals.
o They roam the Kalahari desert with their bows and poisoned arrows, spears, traps and snares.
o They either wear a loin cloth or go virtually naked.
o They travel in small family groups, and live together in open sherms.
o They gather dew from leaves early in the morning and store in ostrich shells.
• The Bindibu:
o The Bindibu or Aborigines of Australia are desert inhabitants and live in very much same way the
Bushman.
o They are lean and dark but healthy.
o They are nomadic hunters and food gatherers, growing no crops and domesticating no animals except
dingo, a wild dog that assists them in tracking down kangaroos, rabbits and birds.
11 www.visionias.in ©Vision IAS

Google it:- https://www.pdfnotes.co


https://www.pdfnotes.co << Download From >> https://
www.pdfnotes.co
o They always stay close to a water supply as they still have not devised a means of tapping and storing
water.
o They live in wurlies, simple shelters made of branches and tufts and grass.
• The Kikuyu:
o The Kikuyu (also known as Agikuyu) are a central Bantu community.
o They share common ancestry with the Embu, Kamba, Tharaka, Meru and Mbeere.
o Traditionally they inhabited the area around Mount Kenya and they are not a desert inhabitant.
o They are among the Kenyan communities that championed the struggle for Kenya’s
independence through the Mau Mau movement and the Kikuyu Central Association (KCA).
o Originally hunters and gatherers, the Agikuyu later adopted agriculture as their main source of
livelihood. Women did the farming and gathering of wild fruits using traditional tools (such as hoes,
digging knives, etc) for domestic consumption, while men did the hunting.
o Today, their main economic activities are trade, agriculture and livestock keeping. They grow many
crops including potatoes, bananas, millet, maize, beans and vegetables. Other common cash crops
grown include tea, coffee and rice.
• Hence option (b) is the correct answer.

Q 33.D
• The process of migration from plain areas to pastures on mountains during summers and again
from mountain pastures to plain areas during winters is known as transhumance.
o In mountain regions, such as the Himalayas, Gujjars, Bakarwals, Gaddis, and Bhotiyas migrate
from plains to the mountains in summers and to the plains from the high altitude pastures in
winters. Hence option (d) is the correct answer.
o Similarly, in the tundra regions, the nomadic herders move from south to north in summers and from
north to south in winters.
• Movement in search of pastures is undertaken either over vast horizontal distances or vertically
from one elevation to another in the mountainous regions.
• Nomadic herding or pastoral nomadism is a primitive subsistence activity, in which the herders rely
on animals for food, clothing, shelter, tools, and transport.
o They move from one place to another along with their livestock, depending on the amount and quality
of pastures and water.
o Each nomadic community occupies a well-identified territory as a matter of tradition.

Q 34.D
• In India, there are different streams of migration. Under the internal migration, four streams are identified:
rural to rural, rural to urban, urban and urban, urban and rural.
• The distribution of male and female migrants in different streams of intra-state and inter-state migration is
presented

• It is clearly evident that females predominate the streams of short distance rural to rural migration in
both types of migration. Most of these were migrants related to marriage. Hence statement 1 is correct.
• Contrary to this, men predominate the rural to an urban stream of inter-state migration due to
economic reasons. Hence statement 2 is correct.
• The reasons for the migration of males and females are different. For example, work and
employment have remained the main cause for male migration (26 per cent) while it is only 2.3 per cent
for females. Contrary to this, about 67 per cent of females move out from their parental houses following
their marriage. Hence statement 3 is correct.
12 www.visionias.in ©Vision IAS

Google it:- https://www.pdfnotes.co


https://www.pdfnotes.co << Download From >> https://
www.pdfnotes.co
Q 35.A
• Recent Context: RBI transfers the “surplus” to the government, in accordance with Section 47
(Allocation of Surplus Profits) of the Reserve Bank of India Act, 1934. Current surplus transfer is in
accordance with the Bimal Jalan committee (committee on the Economic Capital Framework of the
Reserve Bank of India) recommendations.
• The panel recommended a clear distinction between the two components of the economic capital of RBI
that is the Realized equity and the Revaluation balances. Realized equity, which is a form of a
contingency fund for meeting all risks or losses primarily built up from retained earnings. It is also
called the Contingent Risk Buffer (CRB). Hence, option (a) is the correct answer.
• Revaluation reserves comprise of periodic marked-to-market unrealized/notional gains/losses in values of
foreign currencies and gold, foreign securities and rupee securities, and a contingency fund.
• The Surplus Distribution Policy of RBI that was finalized is in line with the recommendations of the
Bimal Jalan committee. The Jalan committee recommended that the size of the realized equity, in the
form of Contingent Risk Buffer (CRB), must be maintained between 5.5% to 6.5% of the RBI’s
balance sheet. Adhering to the recommendations, the RBI has decided to set the CRB level at 5.5% (the
lower limit) of the balance sheet, while transferring the remaining excess to the government.

Q 36.A
• The Jal Kranti Abhiyan launched by the Government of India in 2015–16 with an aim to ensure water
security through per capita availability of water in the country.
• The Jal Kranti Abhiyan aims at involving local bodies, NGOs, and citizens, at large, in creating awareness
regarding its objectives. The following activities have been proposed under the Jal Kranti Abhiyan:
o Selection of one water-stressed village in every 672 districts of the country to create a ‘Jal
Gram’. Hence option(a) is not correct.
o Ídentification of the model command area of about 1000 hectares in different parts of the
country, for example, UP, Haryana (North), Karnataka, Telangana, Tamil Nadu (South),
Rajasthan, Gujarat (West), Odisha (East), Meghalaya (North-East).
o Abatement of pollution:
§ Water conservation and artificial recharge.
§ Reducing groundwater pollution.
§ Construction of Arsenic-free wells in selected areas of the country
o Creating mass awareness through social media, radio, TV, print media, poster and essay writing
competitions in schools.
• Jal Kranti Abhiyan is designed to provide livelihood and food security through water security.

Q 37.C
• The basic principle behind this type of farming is based on social ownership of the means of production
and collective labor. Collective farming or the model of Kolkhoz was introduced in the erstwhile Soviet
Union to improve upon the inefficiency of the previous methods of agriculture and to boost agricultural
production for self-sufficiency. The farmers pool in all their resources like land, livestock, and labor.
However, they are allowed to retain very small plots to grow crops in order to meet their daily
requirements. Yearly targets are set by the government and the produce is also sold to the state at fixed
prices. Produce in excess of the fixed amount is distributed among the members or sold in the market. The
farmers have to pay taxes on the farm produces, hired machinery, etc. Members are paid according to the
nature of the work allotted to them by the farm management. Exceptional work is rewarded in cash or
kind. This type of farming was introduced in the former Soviet Union under the socialist regime which
was adopted by the socialist countries. After its collapse, these have already been modified.
• Hence option (c) is the correct answer.

Q 38.B
• The human development index (HDI) ranks the countries based on their performance in the key areas of
health, education, and access to resources. These rankings are based on a score between 0 to 1 that a
country earns from its record in the key areas of human development. Hence statement 1 is not correct.
• The indicator chosen to assess health is life expectancy at birth. A higher life expectancy means that
people have a greater chance of living longer and healthier lives. Hence statement 2 is not correct.
• The adult literacy rate and the gross enrolment ratio represent access to knowledge. The number of
adults who are able to read and write and the number of children enrolled in schools show how easy or
difficult it is to access knowledge in a particular country. Access to resources is measured in terms of
purchasing power (in U.S. dollars).
13 www.visionias.in ©Vision IAS

Google it:- https://www.pdfnotes.co


https://www.pdfnotes.co << Download From >> https://
www.pdfnotes.co
• Each of these dimensions is given a weightage of 1/3. The human development index is a sum total of
the weights assigned to all these dimensions. The closer a score is to one, the greater is the level of
human development. Therefore, a score of 0.983 would be considered very high while 0.268 would mean
a very low level of human development.
• The human development index measures attainments in human development. It reflects what has been
achieved in the key areas of human development. Yet it is not the most reliable measure. This is because it
does not say anything about the distribution.
• Pakistani economist Mahbub ul Haq created HDI in 1990 which was further used to measure the
country's development by the United Nations Development Program (UNDP). Every year UNDP
ranks countries based on the HDI report released in their annual report on Human Development.
Hence statement 3 is correct
Q 39.D
• Since exploration for oil began during the early 1860s, some 50,000 oil fields have been discovered. More
than 90 percent of these fields are insignificant in their impact on world oil production. The two largest
classes of fields are the super-giants, fields with 5 billion or more barrels of ultimately recoverable oil,
and world-class giants, fields with 500 million to 5 billion barrels of ultimately recoverable oil. Fewer
than 40 supergiant oil fields have been found worldwide.
• The Arabian-Iranian sedimentary basin in the Persian Gulf region contains two-thirds of these supergiant
fields. The remaining super-giants are distributed as follows: two in the United States, two in Russia, two
in Mexico, one in Libya, one in Algeria, one in Venezuela, and two in China.
• The nearly 280 world-class giant fields thus far discovered, plus the super-giants, account for about 80
percent of the world’s known recoverable oil. There are, in addition, approximately 1,000 known large oil
fields that initially contained between 50 million and 500 million barrels. These fields account for some
14 to 16 percent of the world’s known oil. Major oil fields are listed below:
o Ghawar field – Saudi Arabia
o Burgan field – Kuwait
o Azeri-Chirag-Guneshli – Caspian Sea, Azerbaijan
o Ku-Maloob-Zaap – Mexico
o Zakum - UAE
o Ferdows field – Iran
o Sugar Loaf field – Brazil
o Bolivar Coastal field – Venezuela
o World’s five largest offshore oilfields:
§ Safaniya oilfield – Persian Gulf, Saudi Arabia
§ Upper Zakum oilfield – Persian Gulf, UAE
§ Manifa oilfield – Persian Gulf, Saudi Arabia
§ Kashagan oilfield – Caspian Sea, Kazakhstan
§ Lula Oilfield – Brazil
o According to current estimates, more than 81% of the world's proven oil reserves are located in OPEC
Member Countries, with the bulk of OPEC oil reserves in the Middle East.
• Hence option (d) is the correct answer.

Q 40.B
• To understand why a particular region keeps reporting low or high levels of human development it is
important to look at the pattern of government expenditure on the social sector.
• The political environment of the country and the amount of freedom people have is also important.
Countries with high levels of human development invest more in the social sectors and are generally free
from political turmoil and instability. Distribution of the country’s resources is also far more
equitable. Hence statements 3 and 4 are correct
• Places with low levels of human development tend to spend more on defence rather than social
sectors. This shows that these countries tend to be located in areas of political instability and have not
been able to initiate accelerated economic development. Hence statement 2 is not correct while
statement 1 is correct.

Q 41.A
• Recent Context: Russia has announced that it would be withdrawing from the International Space Station
in 2025, and build and manage its own floating laboratory that will be launched into orbit by 2030.
• The Mir space station of the former Soviet Union, and later operated by Russia, was functional from 1986
to 2001. The ISS has been in space since 1998.
14 www.visionias.in ©Vision IAS

Google it:- https://www.pdfnotes.co


https://www.pdfnotes.co << Download From >> https://
www.pdfnotes.co
• The International Space Station (ISS) is Earth’s microgravity laboratory that has allowed more
than 3,600 researchers in 106 countries to conduct more than 2,500 experiments – and the research
continues. Hence, statement 1 is correct.
• The station serves as space environment research laboratory in which scientific research is conducted in
astrobiology, astronomy, meteorology, physics, and other fields. The ISS is suited for testing the
spacecraft systems and equipment required for possible future long-duration missions to the Moon and
Mars.
• It is a joint initiative between the five participating space agencies that have been running it: NASA
(United States), Roscosmos (Russia), JAXA (Japan), ESA (Europe), and CSA (Canada). Hence,
statement 3 is not correct.
• The ownership and use of the space station is established by intergovernmental treaties and agreements.
• The space station orbits in Low Earth Orbit at an average altitude of 227 nautical miles/420
kilometers above Earth. Hence, statement 2 is correct.
• The ISS travels at about 17,500 miles/28,000 kilometers per hour. At this speed, the ISS orbits the Earth
every 90 minutes, which gives the crew 16 sunrises and sunsets every day. Since humans have been living
and working on the space station, it has orbited Earth tens of thousands of times.
• An astronaut's primary job while on the space station is to conduct scientific experiments and maintain the
space station.

Q 42.C
• With a long coastline of 7,516.6 km, India is dotted with 12 major and 200 notified non-
majors (minor/intermediate) ports. These major ports handle 95 per cent of India’s foreign trade.
• Kandla in Kuchchh in Gujarat was the first port developed soon after Independence. It is also known as
the Deendayal Port. Located on the Gulf of Kutch, it is one of major ports on west coast. Kandla was
constructed in the 1950s as the chief seaport serving western India, after the partition of India from
Pakistan left the port of Karachi in Pakistan.
• Marmagao port in Goa is the premier iron ore exporting port of the country.
• Paradip port located in Odisha, specialises in the export of iron ore. Late Biju Patnaik, the then Chief
Minister of Odisha, is the founder father of Paradip Port. It is the only Major Port in the State of Odisha
situated 210 nautical miles south of Kolkata and 260 nautical miles north of Visakhapatnam on the east
coast on the shore of Bay of Bengal.
• Hence option (c) is the correct answer.

Q 43.A
• The Hausa, settled cultivators:
o The Hausa are a tribe of settled cultivators who inhabit the savanna-lands of the Bauchi Plateau of
northern Nigeria.
o They number almost six million and have been organised in settled agricultural communities for
hundreds of years.
o They are more advanced in their civilisation and ways of life than many of the other African tribes.
o They live in towns or villages.
o They do not practise shifting cultivation as many tribes do. Instead, they clear a piece of land and
use it for several years, growing a wide range of crops like maize, millet, groundnuts etc. They also
cultivate non-food crops like cotton and tobacco.
o Besides cultivation, the Hausa also make use of domesticated animals, but they are the only
subsidiary to crop cultivation.
• Nomadic Herding (pastoral nomadism):
o Nomadic herding is a primitive subsistence activity.
o They move from place to place along with their livestock, depending on the availability of pastures &
water.
o A wide variety of animals is kept in different regions.
o In tropical Africa, cattle are the most important livestock, while in Sahara and Asiatic deserts, sheep,
goats and camel are reared.
o In the mountainous areas of Tibet and Andes, yak and llamas and in the Arctic and sub-Arctic regions,
reindeer are the most important animals.
o Pastoral nomadism is associated with three important regions.
§ The core region extends from the Atlantic shores of North Africa eastwards across the Arabian
Peninsula into Mongolia and Central China.
§ The second region extends over the tundra region of Eurasia.
15 www.visionias.in ©Vision IAS

Google it:- https://www.pdfnotes.co


https://www.pdfnotes.co << Download From >> https://
www.pdfnotes.co
§ In the southern hemisphere, there are small areas in South-west Africa and on the island of
Madagascar.
o The Bedouin of Arabian Desert:
§ They are nomadic herdsmen.
§ All round the year, the Bedouin wander with their herds in search of water and green pastures.
They follow a regular pattern of routes, along which are scanty patches of pastures, wells or
springs.
§ They ride on horses and live in tents.
§ They are the best examples of a desert group who have fared well as nomadic herdsmen.
§ Their wealth is their animals.
§ They are also engaged in trade with the caravan merchants and oases people.
o The Kirghiz of the Asiatic Steppes:
§ They used to travel long distances, in search of grass and water.
§ From domesticated animals, they obtained meat, milk, wool, hides, bones and horn.
§ The harsh environment, with long droughts and unreliable showers, made the Kirghiz a tough and
fearless people, ‘the Tartars’, and they long resisted the subjugation of the Russians.
o The Chukchi of north-eastern Asia:
§ They are nomadic people.
§ They are ancient Arctic people who chiefly live on the Chukchi peninsula or Chukotka.
> The territory is mostly tundra (treeless arctic plains), with some taiga areas (plains with
scattered trees) in the south.
§ The Chukchi language belongs to the Paleoasiatic language family.
§ They were one of the last Siberian peoples to fall under Russian rule.
• Hence option (a) is the correct answer.

Q 44.A
• In India apart from internal migration, there is large scale migration from neighbouring countries.
Migration from different countries is given in the following table:

• Bangladesh leads the neighbouring countries in terms of the number of migrants to India from it followed
by Pakistan, Nepal and Sri Lanka. Hence option (a) is the correct answer.

Q 45.B
• Mixed farming is a type of farming that involves both the growing of crops and the raising of livestock.
Equal emphasis is laid on crop cultivation and animal husbandry. Animals like cattle, sheep, pigs, and
poultry provide the main income along with crops.
• This form of agriculture is found in the highly developed parts of the world, e.g. North-western Europe,
Eastern North America, parts of Eurasia, and the temperate latitudes of Southern continents.
• Mixed farms are moderate in size and usually, the crops associated with them are wheat, barley, oats, rye,
maize, fodder, and root crops. Fodder crops are an important component of mixed farming. Crop
16 www.visionias.in ©Vision IAS

Google it:- https://www.pdfnotes.co


https://www.pdfnotes.co << Download From >> https://
www.pdfnotes.co
rotation and intercropping play an important role in maintaining soil fertility. Hence, statement 2 is
correct.
• Mixed farming is characterized by high capital expenditure on Agri inputs such as farm machinery
and building, extensive use of chemical fertilizers and green manures, and also the skill and expertise of
the farmers. Hence, statement 1 is not correct.

Q 46.A
• India produces about 16.6 percent of the total groundnut production in the world (2016). It is
largely a rainfed Kharif crop of drylands. But in southern India, it is cultivated during the rabi
season as well. It covers about 3.6 percent of the total cropped area in the country. Gujarat,
Rajasthan, Tamil Nadu, Telangana, Andhra Pradesh, Karnataka, and Maharashtra are the leading
producers. The yield of groundnut is comparatively high in Tamil Nadu where it is partly irrigated. But
its yield is low in Telangana, Andhra Pradesh, and Karnataka. Hence option (a) is the correct answer.
• Rapeseed and mustard comprise several oilseeds as rai, Sarson, toria, and taramira. These are
subtropical crops cultivated during the rabi season in north-western and central parts of
India. These are frost-sensitive crops and their yields fluctuate from year to year. But with the expansion
of irrigation and improvement in seed technology, their yields have improved and stabilized to some
extent. About two-thirds of the cultivated area under these crops is irrigated. These oilseeds together
occupy only about 2.5 percent of the total cropped area in the country. Rajasthan contributes about one-
third of production while other leading producers are Haryana and Madhya Pradesh. Yields of these crops
are comparatively high in Haryana and Rajasthan.
• Gram is cultivated in subtropical areas. It is mostly a rainfed crop cultivated during the rabi season
in central, western, and northwestern parts of the country. Just one or two light showers or
irrigations are required to grow this crop successfully. At present, gram covers only about 2.8 percent
of the total cropped area in the country. Madhya Pradesh, Uttar Pradesh, Maharashtra, Andhra Pradesh,
Telangana, and Rajasthan are the main producers of this pulse crop. The yield of this crop continues to be
low and fluctuates from year to year even in irrigated areas.

Q 47.C
• Crop cultivation in equatorial regions:
o Natural rubber was first discovered in the Amazon basin, it has since been transplanted to other
parts of the equatorial lands and is grown very profitably on large estates. Malaysia and
Indonesia are the leading producers, each accounting for more than a third of the world's production.
The home country, Brazil exports practically no natural rubber.
o Cocoa is most extensively cultivated in West Africa, bordering the Gulf of Guinea. The two most
important producers are Ghana and Nigeria. All the cocoa here goes into the American and
European chocolate industries.
o Oil palm is another important crop. Large areas of tropical forests and other ecosystems with high
conservation values have been cleared to make room for vast monoculture oil palm plantations.
Around 90% of the world's oil palm trees are grown on a few islands in Malaysia and Indonesia –
islands with the most biodiverse tropical forests found on Earth.
o Other crops that have been found suitable for the hot, wet equatorial climate and are extensively
cultivated are coconuts, sugar, coffee, tea, tobacco, spices, cinchona, bananas, pineapples, and
sago.
o Cotton is a tropical crop and it is raised in India as a Kharif crop. Cotton requires uniformly high
temperatures between 21 degrees Celsius and 30 degrees Celsius. During October, the day
temperature should be above 26 degrees Celsius which helps the ripening and bursting of cotton balls
under the sunny skies.
o Hence option (c) is the correct answer.

Q 48.C
• The double village settlements spread on both sides of a river where there is bridge or a ferry.
• It is a group of two settlement unit grown up simultaneously or one after another at a place. In this village
pattern nallah, stream, river act as a boundary between the two settlements. These villages occupy similar
geographic conditions but their revenue and administration dealt separately.
• Hence option (c) is the correct answer.

17 www.visionias.in ©Vision IAS

Google it:- https://www.pdfnotes.co


https://www.pdfnotes.co << Download From >> https://
www.pdfnotes.co
Q 49.B
• The ancient Indian scriptures were concerned about the balance and harmony among the elements of
nature. Mahatma Gandhi advocated for harmony and balance between population, resources, and
development.
• These views related to Sustainable development were also re-echoed in the Club of Rome Report
“Limits to Growth” (1972), Schumacher’s book “Small is Beautiful” (1974), Brundtland
Commission’s Report “Our Common Future” (1987), and finally in the “Agenda-21 Report of the Rio
Conference” (1993). Hence option (b) is the correct answer.
• Additional Information:
o The Limits to Growth is a 1972 report on the exponential economic and population growth with a
finite supply of resources, studied by computer simulation. It was commissioned by the Club of
Rome. The message of this book still holds today: The earth’s interlocking resources – the global
system of nature in which we all live – probably cannot support present rates of economic and
population growth much beyond the year 2100, if that long, even with advanced technology.

Q 50.A

18 www.visionias.in ©Vision IAS

Google it:- https://www.pdfnotes.co


https://www.pdfnotes.co << Download From >> https://
www.pdfnotes.co

• Hence option (a) is the correct answer.

Q 51.A
Distribution of Minerals
• Asia: China and India have large iron ore deposits. The continent produces more than half of the world’s
tin. China, Malaysia, and Indonesia are among the world’s leading tin producers. China also leads in the
production of lead, antimony, and tungsten. Asia also has deposits of manganese, bauxite, nickel, zinc,
and copper.
• Europe: Europe is the leading producer of iron-ore in the world. The countries with large deposits of iron
ore are Russia, Ukraine, Sweden, and France. Minerals deposits of copper, lead, zinc, manganese, and
nickel are found in eastern Europe and European Russia.
• North America: The mineral deposits in North America are located in three zones:
o the Canadian region north of the Great Lakes,
o the Appalachian region and
o the mountain ranges of the west.
• Iron ore, nickel, gold, uranium, and copper are mined in the Canadian Shield Region, coal in the
Appalachian region. Western Cordilleras have vast deposits of copper, lead, zinc, gold, and silver.
• South America: Brazil is the largest producer of high-grade iron ore in the world. Chile and Peru are
leading producers of copper. Brazil and Bolivia are among the world’s largest producers of tin. South
America also has large deposits of gold, silver, zinc, chromium, manganese, bauxite, mica, platinum,
asbestos, and diamond. Mineral oil is found in Venezuela, Argentina, Chile, Peru, and Columbia.
• Africa: Africa is rich in mineral resources. It is the world’s largest producer of diamonds, gold, and
platinum. South Africa, Zimbabwe, and Zaire produce a large portion of the world’s gold. The other
minerals found in Africa are copper, iron ore, chromium, uranium, cobalt, and bauxite. Oil is found in
Nigeria, Libya, and Angola. Hence, statement 2 is correct.
• Australia: Australia is the largest producer of bauxite in the world. It is a leading producer of gold,
diamond, iron ore, tin, and nickel. It is also rich in copper, lead, zinc, and manganese. Kalgoorlie and
Coolgardie areas of western Australia have the largest deposits of gold. Hence, statement 1 is correct.
• Antarctica: The geology of Antarctica is sufficiently well known to predict the existence of a variety of
mineral deposits, some probably large. The significant size of deposits of coal in the Transantarctic
Mountains and iron near the Prince Charles Mountains of East Antarctica is forecasted. Iron ore, gold,
silver, and oil are also present in commercial quantities. Hence, statement 3 is not correct.

Q 52.C
• The Yakuts:
o The Yakuts (or Sakha, as they call themselves) are a native Siberian population that mainly live
throughout the Sakha Autonomous Republic (Yakutia) of the Russian Federation. Hence pair 1 is not
correctly matched.
19 www.visionias.in ©Vision IAS

Google it:- https://www.pdfnotes.co


https://www.pdfnotes.co << Download From >> https://
www.pdfnotes.co
o Traditional Yakut economy is based on cattle and horse breeding that is similar to the form of
pastoralism practised by various Turkic and Mongolic groups inhabiting the Asian steppes.
• The Samoyeds:
o They speak Samoyedic languages, which are part of the Uralic family. They are a linguistic grouping,
not an ethnic or cultural one. The name derives from the obsolete term Samoyed used in Russia for
some indigenous peoples of Siberia. Hence pair 2 is correctly matched.
• The Tuaregs:
o The Tuaregs of the Sahara are camel riders and dwell in grass zeriba. Hence pair 3 is not correctly
matched.
o The Tuaregs are part of the Berber group of people, and they are largely Muslim. Much of Tuareg art
is in the form of jewellery, leather and metal saddle decorations, and finely crafted swords.
o The Tuaregs are kind of like the Kurds of the Middle East.
o They are a substantial ethnic population that lives across the boundaries of several countries but has
no majority in any one country.

Q 53.B
• The current population of the world is nearing 8 billion. It has grown to this size over centuries. In the
early periods' population of the world grew very slowly. It is only during the last few hundred years that
the world population has increased at an alarming rate.
• After the evolution and introduction of agriculture, the population began to grow slowly but steadily.
Around 1750, at the dawn of the Industrial Revolution, the world population was 550 million.
• The world population exploded in the eighteenth century after the Industrial Revolution. Technological
advancement achieved so far helped in the reduction of the birth rate and provided a stage for accelerated
population growth.
• As a consequence, the world population reached 1 billion in the early 19th century. Thus, it took
millions of years for the human population to reach the 1 billion mark. In contrast, it took only 12
years for the world population to reach 7 billion from 6 billion. Hence, statement 1 is not correct
• The growth rates of the continents of the world are:
o Africa - 2.49 %
o Oceania - 1.31% (Australia, New Zealand, and other island nations)
o Asia - 0.86%
o South America - 0.83%
o North America - 0.77%
o Europe - 0.06%
• At 2.49 per cent, Africa has the highest growth rate of population among all the continents. Hence,
statement 2 is correct.

Q 54.D
• Recently the report titled Protected Planet Report 2020 is released by United Nations Environment
Programme (UNEP) and the International Union for the Conservation of Nature (IUCN). Hence
option (d) is the correct answer.
• This report underlined the progress the world has made toward the ambitious goals agreed by countries in
2010 at the United Nations Convention on Biological Diversity.
• The 2020 edition provides the final report on the status of Aichi Biodiversity Target 11(to conserve 17 per
cent of land and inland water ecosystems and 10 per cent of its coastal waters and oceans by 2020,) and
looks to the future as the world prepares to adopt a new post-2020 global biodiversity framework.
• According to the findings of the report:
o Since 2010, protected areas covering almost 21 million km2 have been added to the global network.
o As many as 82% of countries and territories increased their share of protected area and coverage of
other effective area-based conservation measures (OECM) since 2010. OECM are a conservation
designation for areas that are achieving the effective in-situ conservation of biodiversity outside of
protected areas.
Q 55.D
• This has been one of the major industrial regions of Europe for a long time. Coal and iron and steel
formed the basis of the economy, but as the demand for coal declined, the industry started shrinking. Even
after the iron ore was exhausted, the industry remained, using imported ore brought by waterways to the
Ruhr. The Ruhr region is responsible for 80 percent of Germany’s total steel production. Hence pair 1 is
correctly matched.
20 www.visionias.in ©Vision IAS

Google it:- https://www.pdfnotes.co


https://www.pdfnotes.co << Download From >> https://
www.pdfnotes.co
• The industry is one of the most complex and capital-intensive industries and is concentrated in the
advanced countries of North America, Europe, and Asia. In the U.S.A, most of the production comes from
the northern Appalachian region (Pittsburgh), Great Lake region (Chicago-Gary, Erie, Cleveland, Lorain,
Buffalo, and Duluth), and the Atlantic Coast (Sparrows Point and Morrisville). The industry has also
moved towards the southern state of Alabama. The Pittsburg area is now losing ground. It has now
become the “rust bowl” of the U.S.A.Hence pair 2 is correctly matched.
• In Europe, U.K., Germany, France, Belgium, Luxembourg, the Netherlands, and Russia are the leading
producers. The important steel centers are Birmingham and Sheffield in the U.K.; Duisburg, Dortmund,
Dusseldorf and Essen in Germany; Le Creusot and St. Ettiennein France; and Moscow, St. Petersburgh,
Lipetsk, Tula, in Russia and Krivoi Rog, and Donetsk in Ukraine. Hence pair 3 is correctly matched.

Q 56.A
• The importance of foodgrains in the Indian agricultural economy may be gauged from the fact these crops
occupy about two-thirds of the total cropped area in the country. Foodgrains are dominant crops in all
parts of the country whether they have subsistence or commercial agricultural economy. On the basis of
the structure of grain, the foodgrains are classified as cereals and pulses.
• The cereals occupy about 54 percent of the total cropped area in India. India produces a variety of
cereals, which are classified as fine grains (rice, wheat) and coarse grains (jowar, bajra, maize, ragi),
etc. Hence statement 1 is correct.
• Rice is a staple food for the overwhelming majority of the population in India. About one-fourth of the
total cropped area in the country is under rice cultivation. West Bengal, Uttar Pradesh, and
Punjab are the leading rice-producing states in the country.
• Wheat is the second most important cereal crop in India after rice. About 14 percent of the total
cropped area in the country is under wheat cultivation. Uttar Pradesh, Madhya Pradesh, Punjab,
Haryana, and Rajasthan are leading wheat-producing states. Hence statement 2 is not correct.

Q 57.D
• The age-sex structure of a population refers to the number of females and males in different age groups. A
population pyramid is a graphical illustration of the distribution of a population by age groups and gender.
• The shape of the population pyramid reflects the characteristics of the population. The left side shows the
percentage of males while the right side shows the percentage of women in each age group.
• There are generally three types of age-sex pyramids
• Expanding Populations
o These form a triangular-shaped pyramid with a wide base. These have larger populations in
lower age groups due to high birth rates. Most of the developing countries have high birth rates
and represent this shape. Examples include countries like Nigeria, Pakistan, etc. Hence
statement 1 is correct.
• Constant Populations
o These form a bell-shaped pyramid that is tapered towards the top. This shows birth and death
rates are almost equal leading to a near-constant population. Hence statement 2 is correct.
• Declining Populations
o These form a pyramid that has a narrow base and a tapered top showing low birth and death
rates. This is typical of developed countries like Japan, Germany. Hence statement 3 is correct.
Q 58.D
• Manganese is not found as a free element in nature. It is often found in combination with iron. It is an
important raw material for smelting iron ore and also used for manufacturing ferroalloys. Hence
statement 1 is correct.
• India possesses the second-largest reserves in the world after Zimbabwe and is the fifth-largest producer.
• Manganese deposits are found in almost all geological formations, however, it is mainly associated
with Dharwar system. Hence statement 2 is correct.
• Odisha is the leading producer of Manganese. Hence statement 3 is correct.
• Major mines in Odisha are located in the central part of the iron ore belt of India, particularly in Bonai,
Kendujhar, Sundergarh, Gangpur, Koraput, Kalahandi, and Bolangir.
• Karnataka is another major producer and here the mines are located in Dharwar, Ballari, Belagavi,
North Canara, Chikkmagaluru, Shivamogga, Chitradurg, a and Tumakuru.
• Maharashtra is also an important producer of manganese, which is mined in Nagpur, Bhandara, and
Ratnagiri districts.
• Telangana, Goa, and Jharkhand are other minor producers of manganese.
21 www.visionias.in ©Vision IAS

Google it:- https://www.pdfnotes.co


https://www.pdfnotes.co << Download From >> https://
www.pdfnotes.co
Q 59.D
• The Northern Atlantic Sea Route links North-eastern U.S.A. and North-western Europe, the two
industrially developed regions of the world. The foreign trade over this route is greater than that of the
rest of the world combined. One-fourth of the world's foreign trade moves on this route. It is, therefore,
the busiest in the world and otherwise, called the Big Trunk Route. Hence option (d) is the correct
answer.
• Both the coasts have highly advanced ports and harbor facilities. Rich agricultural, commercial, and
industrial regions of Europe export large quantities of manufactured items-textiles, chemicals, machinery,
fertilizers, steel, and wine to the United States and Canada. Bulky and large quantities of food grains and
raw materials, like wheat, wood pulp, copper as well as iron and steel, transport, copper as well as iron
and steel, transport, copper as well as iron and steel. Transport equipment etc. is sent to Western Europe
through this route.

Q 60.D
• The first lunar eclipse of 2021 took place on 26th May 2021. It was a superlunar event, as it will be a
supermoon, a lunar eclipse and a red blood moon all at once.
• It was termed as super blood moon, a combination of a total lunar eclipse and a brighter-than-usual super
moon.
• Breaking down the different terms of this event.
o A supermoon occurs when a full or new moon coincides with the moon’s closest approach to the
Earth (perigee). Hence option 2 is correct.
o When sun and moon are on the same side of earth, a new moon appears. When sun and moon are on
the opposite side of the earth, a full moon appears. A lunar eclipse can only happen at full
moon. Hence option 3 is correct.

o A "blood moon" happens when Earth's moon is in a total lunar eclipse.


§ A total lunar eclipse takes place when the Earth comes between the Sun and the Full Moon
and blocks the Sun's direct rays from lighting up the Moon i.e. Moon must lie within the
Umbra region of Earth. Hence option 1 is correct.

22 www.visionias.in ©Vision IAS

Google it:- https://www.pdfnotes.co


https://www.pdfnotes.co << Download From >> https://
www.pdfnotes.co

• The umbra and penumbra are distinct parts of a shadow, created by any light source after impinging on an
opaque object

Q 61.B
• The reasons for migration can be put into two broad categories :
o push factor, these cause people to leave their place of residence or origin; and
o pull factors, which attract people from different places.
• In India, people migrate from rural to urban areas mainly due to poverty, high population pressure on
the land, lack of basic infrastructural facilities like health care, education, etc. Apart from these
factors, natural disasters such as floods, drought, cyclonic storms, earthquakes, tsunami, wars, and local
conflicts also give an extra push to migrate. Hence option (b) is the correct answer.
• The most important pull factor for a majority of the rural migrants to urban areas is the better
opportunities, availability of regular work, and relatively higher wages. Better opportunities for
education, better health facilities and sources of entertainment, etc., are also quite important pull factors.

Q 62.C
• Watershed management basically refers to the efficient management and conservation of surface
and groundwater resources. It involves the prevention of runoff and storage and recharge of
groundwater through various methods like percolation tanks, recharge wells, etc.
• However, in broad sense watershed management includes conservation, regeneration, and judicious
use of all resources – natural (like land, water, plants, and animals) and humans within a
watershed. Hence statement 1 is correct.
• Watershed management aims at bringing about a balance between natural resources on the one hand and
society on the other. The success of watershed development largely depends upon community
participation. Hence statement 2 is correct.
Q 63.D
• Recent Context: Recent researches have reported weathering of Pleistocene-era rock paintings dating
back to 45,000-20,000 years ago in cave sites on the Indonesian island of Sulawesi.
o The artwork in the area includes what is believed to be the world’s oldest hand stencil (almost 40,000
years ago), created by pressing the hand on a cave wall, and spraying wet red-mulberry pigments over
it. The artwork made with pigments was decaying due to a process known as haloclasty, which
is triggered by the growth of salt crystals due to repeated changes in temperature and humidity,
caused by alternating wet and dry weather in the region.
• Haloclasty is a type of physical weathering caused by the growth of salt crystals. The process is first
started when saline water seeps into cracks and evaporates depositing salt crystals. When the rocks are
then heated, the crystals will expand putting pressure on the surrounding rock which will over time
splinter the stone into fragments. Hence, option (d) is the correct answer.
• Salt crystallization may also take place when solutions decompose rocks (for example, limestone and
chalk) to form salt solutions of sodium sulfate or sodium carbonate, from which water evaporates to
form their respective salt crystals.
23 www.visionias.in ©Vision IAS

Google it:- https://www.pdfnotes.co


https://www.pdfnotes.co << Download From >> https://
www.pdfnotes.co
• The salts which have proved most effective in disintegrating rocks are sodium sulfate, magnesium sulfate,
and calcium chloride. Some of these salts can expand up to three times or more in volume.
• It is normally associated with arid climates where strong heating causes strong evaporation and
therefore salt crystallization. It is also common along coasts. An example of salt weathering can be seen
in the honeycombed stones in sea walls.
• One key benefit of haloclasty, and weathering in general, is the formation of soil, which is crucial for the
growth of plants. Without the process, many parts of the Earth would be an uninhabitable pile of rocks.

Q 64.C
• The density of population is defined as the ratio of the total number of people per unit area of land. It is
a crude measure of human and land relationships. To get a better insight into the human-land ratio in
terms of pressure of population on total cultivable land, the physiological and the agricultural densities
should be found out which are significant for a country like India having a large agricultural population.
• Physiological density refers to the number of people per unit area of net cultivable land. It is a basic
indicator of a country's food-producing capability and the human pressures placed upon it.
o Physiological density = total population / net cultivated area
o Hence option (c) is the correct answer.
• Agricultural density refers to the total number of agricultural population per unit area of net cultivable
area. Here Agricultural population includes cultivators and agricultural labourers and their family
members.
o Agricultural density = total agricultural population / net cultivable area

Q 65.C
• The distribution of population is highly skewed in the world. It depends on various factors such as
o Geographical Factors
o Economic Factors
o Socio-Cultural Factors
• River valleys come under geographical factors which tend to increase the density of population due to its
water availability and generally good fertile soils.
• Hilly areas are sparsely populated due to their inaccessibility and their difficulty in practising
agriculture etc.
• Mineral-rich areas attract industries and generate employment opportunities. This provides a strong pull
factor for people and makes such areas densely populated.
• Urbanization: Cities offer better employment opportunities, educational and medical facilities, better
means of transport and communication. Good civic amenities and the attraction of city life draw people to
the cities. It leads to rural to urban migration and the population density increases.
• Hence option (c) is the correct answer.

Q 66.A
• Petroleum is also called ‘black gold’ or ‘liquid gold. It is an essential source of energy for all internal
combustion engines in automobiles.
• Crude petroleum occurs in sedimentary rocks of the tertiary period. It is formed when large
quantities of dead organisms, usually zooplankton and algae, are buried underneath sedimentary rock and
subjected to intense heat and pressure. Hence, statement 1 is correct.
• Unlike coal, Petroleum is not distributed evenly around the world. More than half of the world’s proven
oil reserves are located in the Middle East. Following the Middle East are Canada and the United
States, Latin America, Africa, and the region occupied by the former Soviet Union. Each of those regions
contains less than 15 percent of the world’s proven reserves. Hence, statement 2 is not correct.
• The two largest classes of fields are the supergiants, fields with 5 billion or more barrels of ultimately
recoverable oil, and world-class giants, fields with 500 million to 5 billion barrels of ultimately
recoverable oil. Fewer than 40 supergiant oil fields have been found worldwide. The Arabian Iranian
sedimentary basin in the Persian Gulf region contains two-thirds of these supergiant fields.

Q 67.D
• The Pygmies:
o They are the extremely short-statured people of the Congo Basin in Africa.
o They are scattered across equatorial Africa, where they speak various languages, inhabit different
types of forests, and hunt and gather food in diverse ways.
24 www.visionias.in ©Vision IAS

Google it:- https://www.pdfnotes.co


https://www.pdfnotes.co << Download From >> https://
www.pdfnotes.co
o They inhabit a narrow band of tropical rain forest about four degrees above and four degrees below
the Equator, stretching from Cameroon's Atlantic coast eastward to Lake Victoria in Uganda.
o They are the largest group of hunter-gatherers left on earth. But they are under serious threat.
• The Eskimos:
o Greenland, northern Canada and Alaska are their lands.
o They used to live as hunters, fishers and food-gatherers but in recent years more and more of them are
settling in permanent huts.
o The Polar Eskimos, living around Thule in north-west Greenland still lead an uncertain life, not
very much different from their forefathers.
o During winter they live in compact igloos and in summer when they move out to hunt they pitch
portable tents of skins by the side of streams
• The Lapps:
o The Lapps are vigorous people of obscure origin who have managed until now to preserve their race,
language, and elements of their culture on the northern fringe of European civilisation.
o The Samis, normally known as Lapps, live in a very wide territory stretching from the coasts of
Norway to the peninsula of Kola, in Russia.
o The Samis are nomadic shepherds and their economy is based on reindeer rearing. Their nomadic
life is due to the reindeers’ dietary needs. These big herbivores mainly feed on slowly-growing
lichens, so they need very large areas to survive.
o The Lapps follow their animals, as they move in search of new pastures. The nomadic groups of
Lapps still live in reindeer skin tents that look like those of native Americans. Traditional Sami dishes
are mainly made of reindeer meat and fish.
• The Masai:
o In the land of the lions, the proud Masai still live the way they did for centuries, with a few significant
changes in their lifestyle.
o They are a nomadic tribe who once wandered with their herds of cattle in the central highlands of
East Africa. But now mainly confined to the few square miles of Masai reserves in Kenya and
Tanzania.
o They build circular huts with sticks, bushes and mud for temporary shelter.
o The cattle kept by the Masai are the zebu cattle with humps and long horns. They are treated with
great respect and affection and are never slaughtered for food or for sale.
o Cattle are kept by every Masai family and are symbols of wealth.
o They drink the blood as well as the milk of their animals but do not kill them for meat.
• Hence option (d) is the correct answer.

Q 68.C
• Migration is a response to the uneven distribution of opportunities over space. People tend to move from a
place of low opportunity and low safety to a place of higher opportunity and better safety. This, in turn,
creates both benefits and problems for the areas, people migrate from and migrate to. Some of the issues
with migration in India are:
• High out-migration from Uttarakhand, Rajasthan, Madhya Pradesh, and Eastern Maharashtra have
brought serious imbalances in age and sex composition in these states. Similar imbalances are also
brought in the recipient's states. Hence statement 1 is correct.
• The development of slums in industrially developed states such as Maharashtra, Gujarat, Karnataka,
Tamil Nadu, and Delhi is a negative consequence of unregulated migration within the country.
• Migration has some serious negative social consequences such as anonymity, which creates a social
vacuum and a sense of dejection among individuals. A continued feeling of dejection may motivate
people to fall into the trap of anti-social activities like crime and drug abuse. Hence statement 2 is
correct.
• Overcrowding of people due to rural-urban migration leads to unplanned growth of urban settlement
and formation of slums shanty colonies. Hence statement 3 is correct.

Q 69.B
• Recent Context: Around Rs 4 crore was recently stolen from the currency chest of Axis Bank in
Chandigarh.
• Currency chest is a place where the Reserve Bank of India (RBI) stocks the money meant for banks
and ATMs. These chests are usually situated on the premises of different banks but administrated
by the RBI.

25 www.visionias.in ©Vision IAS

Google it:- https://www.pdfnotes.co


https://www.pdfnotes.co << Download From >> https://
www.pdfnotes.co
• The RBI offices in various cities receive notes from note presses and coins from the mints. These are sent
to the currency chests and small coin depots from where they are distributed to bank branches. The RBI
has set up over 4,075 currency chests all over the country. Besides these, there are around 3,746 bank
branches that act as small coin depots to stock small coins.
• The money present in the currency chest belongs to the RBI and the money, kept in the strong room
outside the currency chest belongs to the bank. Hence statement 1 is not correct.
• As per the set guidelines, the bank, in which the currency chest is situated is liable to fulfill the loss of the
currency chest.
• The security of currency chests is the subject of the bank in which chests are situated. The Reserve
Bank of India (RBI) reimburses the security expenses to the bank as per the set norms. The expenses
include the cost of transportation of cash from one bank to another. Hence statement 2 is correct.

Q 70.C
• National Waterway includes Kottapuram - Kollam stretch. It includes 168 km of the west coast canal
along with the Champakara canal (14 km) and the Udyogmandal canal (23 km). Its path is as follows

• Hence option (c) is the correct answer.

Q 71.A
• Minerals are usually found in “ores”. The term ore is used to describe an accumulation of any mineral
mixed with other elements. The mineral content of the ore must be in sufficient concentration to make its
extraction commercially viable.
• Minerals generally occur in these forms:
o In igneous and metamorphic rocks minerals may occur in the cracks, crevices, faults, or joints. The
smaller occurrences are called veins and the larger is called lodes. In most cases, they are formed
when minerals in liquid/ molten and gaseous forms are forced upward through cavities towards the
earth’s surface. Major metallic minerals like tin, copper, zinc, and lead, etc. are obtained from
veins and lodes. Hence, statement 1 is correct.
o In sedimentary rocks, a number of minerals occur in beds or layers. They have been formed as a result
of deposition, accumulation, and concentration in horizontal strata. Coal has been concentrated as a
result of long periods under great heat and pressure. Hence, statement 2 is not correct.
o Another mode of formation involves the decomposition of surface rocks, and the removal of soluble
constituents, leaving a residual mass of weathered material containing ores. Bauxite is formed this
way. Hence, statement 3 is not correct.
o Certain minerals may occur as alluvial deposits in sands of valley floors and the base of hills. These
deposits are called ‘placer deposits’ and generally contain minerals, which are not corroded by water.
Gold, silver, tin, and platinum are the most important among such minerals.
26 www.visionias.in ©Vision IAS

Google it:- https://www.pdfnotes.co


https://www.pdfnotes.co << Download From >> https://
www.pdfnotes.co
o The ocean waters contain vast quantities of minerals, but most of these are too widely diffused to be
of economic significance. However, common salt, magnesium, and bromine are largely derived from
ocean waters. The ocean beds, too, are rich in manganese nodules

Q 72.B
• Ageing population is one where the proportion of older people is increasing. This is also known as
'demographic ageing' and 'population ageing.
• In most of the developed countries, the population in higher age groups has increased due to
increased life expectancy and reduced birth rates. Hence, developed countries are experiencing
population ageing.
• In developing countries, the birth rate is still high and life expectancy has not reached the levels of
developed countries. For this reason, most of the developing countries' population is considered
young. Hence, statement 1 is not correct.
• In the last few decades, developed countries experienced a reduction in birth rates due to various
socio-economic reasons like
o Women's education
o Better healthcare facilities
o Nuclear family
o Accessibility of contraceptives
o Economic costs in raising a child, etc
• As a result, the proportion of children in their population has declined. Examples include countries
like Japan, Italy, Germany, etc. Hence, statement 2 is correct

Q 73.D
• Mediterranean Agriculture: The term ‘Mediterranean agriculture’ applies to the agriculture done in
those regions which are having the Mediterranean type of climate. Mediterranean agriculture is unique
because it is a mixture of diverse bio-cultural activities (both animal husbandry and crop farming) that has
developed in five major world regions. This type of agriculture is determined by climatic conditions,
which exert such an influence that both traditional and commercial agriculture flourish with a dominance
of the agriculture of citrus fruits along with horticulture and floriculture.
o Characteristics:
§ The natural setting that lends itself to Mediterranean agriculture is distinguished by erratic
rainfall, mild temperatures, irregular topography, and nearness to large water bodies.
§ In this type, farming is intensive, highly specialized, and varied in the kinds of crops raised.
Subsistence agriculture occurs side by side with commercial farming. Many crops such as wheat,
barley, and vegetables are raised for domestic consumption, while others like citrus fruits, olives,
and grapes are mainly for export. The Mediterranean lands are also known as ‘orchard lands of
the world’.
§ The land use in these areas is dependent on such factors as the total annual amount of rainfall,
length of summer drought, availability of melting snow, local soil conditions, and price
fluctuations in local and world markets.
o The main aspects of Mediterranean agriculture are:
o Orchard fanning: It represents a highly specialized commercial agriculture here. The world supply
of citrus fruits, olives, and figs comes almost exclusively from Mediterranean lands. Fruit culture
has long been a traditional Mediterranean occupation because of the very special climatic features in
Mediterranean areas. Hence statement 1 is correct.
o Viticulture: Viticulture or grape cultivation is a specialty of the Mediterranean region. It represents a
very intensive form of farming requiring not only good conditions of moisture, temperature, and soil
but also much personal care if the grapes are to be of high quality. Hence statement 2 is correct.
o Cereal and vegetable cultivation:
§ In acreage, cereal crops are the most important in Mediterranean agriculture. Wheat,
especially hard winter wheat, is the principal food grain, and barley is grown in the poorer
areas. Hence statement 3 is not correct.
§ In most Mediterranean countries cereals often occupy about half the total cultivated acreage and
provide enough grain for home consumption. The warm and sunny Mediterranean climate also
allows a wide range of other food crops and green vegetables to be harvested.
o Limited animal husbandry:
§ Mediterranean agriculture is also characterised by limited animal husbandry, which survives on
grasslands available here. In areas like Lombardy plain, Ebro basin, San Joaquin valley of
27 www.visionias.in ©Vision IAS

Google it:- https://www.pdfnotes.co


https://www.pdfnotes.co << Download From >> https://
www.pdfnotes.co
California, dairy farming is important. In mountain areas, the practice of ‘transhumance’, moving
the cattle up to mountain pastures in the summer and returning them to the valleys in winter, is
very common.
Q 74.A
• The HVJ (Hazira-Vijaipur-Jagdishpur) Natural Gas Pipeline is India's first cross-state gas pipeline. The
project began in1986, following the formation of the Gas Authority of India Limited (GAIL), to supply
gas to fertilizer companies in Uttar Pradesh. The pipeline runs from Hazira, Gujarat through
Vijaipur, Madhya Pradesh to Jagdishpur, Uttar Pradesh.
• It passes through the states of Gujarat, Madhya Pradesh , Rajasthan, Uttar Pradesh, Haryana, and
Delhi NCT.

Q 75.B
• The number of women and men in a country is an important demographic characteristic. The ratio
between the number of women and men in the population is called the Sex Ratio.
• It is calculated as the number of males per thousand females in many countries. In India, it is calculated as
number of females per thousand males.
• As per the UN (World Population Prospects 2019) the sex ratio of the world is approximately 1016
males per 1000 females. Hence, the world's sex ratio is not favourable to females. Hence, statement
1 is not correct.
• Around 125 countries have more females per thousand males. Hence, statement 2 is correct.
28 www.visionias.in ©Vision IAS

Google it:- https://www.pdfnotes.co


https://www.pdfnotes.co << Download From >> https://
www.pdfnotes.co
• Qatar has the world's worst sex ratio while Nepal has the world's best sex ratio. In 2020, male to female
ratio for Nepal was 84.55 males per 100 females. Sex ratio at birth (male births per female births) in Qatar
was reported at 1.045 in 2019.

Q 76.A
• Pulses are a very important ingredient of vegetarian food as these are rich sources of proteins. These are
legume crops that increase the natural fertility of soils through nitrogen fixation. India is a leading
producer of pulses in the world. Hence statement 1 is correct.
• The cultivation of pulses in the country is largely concentrated in the drylands of Deccan and
central plateaus and northwestern parts of the country. Pulses occupy about 11 percent of the total
cropped area in the country. Hence statement 2 is not correct.
• Being the rainfed crops of drylands, the yields of pulses are low and fluctuate from year to year. Gram and
tur are the main pulses cultivated in India.

Q 77.A
• Access to knowledge about the society and environment are fundamental to development. Literacy is the
beginning of access to such a world of knowledge and freedom.
• The working definition of literacy in the Indian census since 1991 is - the total percentage of the
population of an area at a particular time aged seven years or above who can read and write with
understanding. Here the denominator is the population aged seven years or more.
• Observation from the Census of India, 2011:
o Overall literacy in India is approximately 74.04 percent (2011). while female literacy is 65.46
percent. Total literacy as well as female literacy is higher than the national average in most of the
states from south India.
o There are wide regional disparities in literacy rates across the states of India. There is a state like
Bihar which has very low (63.82 percent) literacy and there are states like Kerala and Mizoram which
have literacy rates.
o In India, the states/UTs which have literacy rates of more than 90% are Kerala (93%), Mizoram
(91%), Lakshadweep Islands (92%). The developed states/UTs like NCT Delhi, Daman & Diu,
Tripura, Chandigarh have literacy rates in the range of 85-90%.
• Hence option (a) is the correct answer.

Q 78.C
• Tertiary activities include both production and exchange. The production involves the ‘provision’ of
services that are ‘consumed’. The output is indirectly measured in terms of wages and salaries. The
exchange involves trade, transport, and communication facilities that are used to overcome distance.
• Tertiary activities, therefore, involve the commercial output of services rather than the production of
tangible goods. They are not directly involved in the processing of physical raw materials. Hence,
statement 1 is not correct.
• The main difference between secondary activities and tertiary activities is that the expertise provided by
services relies more heavily on specialized skills, experience, and knowledge of the workers rather than
on the production techniques, machinery, and factory processes.
• Quaternary activities involve some of the following: the collection, production, and dissemination of
information or even the production of information. Quaternary activities center around research,
development and may be seen as an advanced form of services involving specialized knowledge and
technical skills.
• Quinary activities are services that focus on the creation, re-arrangement, and interpretation of new and
existing ideas; data interpretation, and the use and evaluation of new technologies. The highest level of
decision-makers or policymakers performs quinary activities. Often referred to as ‘gold collar’
professions, they represent another subdivision of the tertiary sector representing special and highly paid
skills of senior business executives, government officials, research scientists, financial and legal
consultants, etc. Hence, statement 2 is not correct and statement 3 is correct.

Q 79.C
• India is a leading country in the world in terms of coffee production and stands 8th among the top 10
coffee-producing countries in the world. There are three varieties of coffee i.e. arabica, robusta, and
liberica. India mostly grows superior quality coffee, arabica, which is in great demand in the
international market.
29 www.visionias.in ©Vision IAS

Google it:- https://www.pdfnotes.co


https://www.pdfnotes.co << Download From >> https://
www.pdfnotes.co
• In India, coffee is mainly produced in Karnataka, Kerala, Tamil Nadu, Andhra Pradesh, and
Odisha.
• Among all coffee-producing states in India, Karnataka is the largest producer and produces more
than 70% of the total coffee produced in the country.
• Kodagu, Chikmagalur, and Hassan are the main coffee-producing regions in Karnataka.
• Hence option (c) is the correct answer.

Q 80.D
• Recent context: A very big iceberg broke off (calved from) the Ronne Ice Shelf in Antarctica, and is
now afloat in the Weddell Sea. It is over 4,300 square kilometers in size and is now the largest iceberg in
the world. The newly calved iceberg, designated A-76 by scientists, was spotted in recent satellite images
captured by the Copernicus Sentinel-1 mission. An iceberg is ice that broke off from glaciers or shelf ice
and is floating in open water.
• The Ronne Ice Shelf (an ice shelf is a floating extension of land ice) on the flank of the Antarctic
Peninsula is one of the largest of several enormous floating sheets of ice that connect to the continent's
landmass and extend out into the surrounding seas.(The Ross Iceshelf is another massive iceshelf in
Antarctica.)
• Hence option (d) is the correct answer.

30 www.visionias.in ©Vision IAS

Google it:- https://www.pdfnotes.co


https://www.pdfnotes.co << Download From >> https://
www.pdfnotes.co
Q 81.C
• Recent Context: Six sites were recently added to India’s tentative list of UNESCO world heritage sites.
• A World Heritage Site is a landmark or area with legal protection by an international convention
administered by the United Nations Educational, Scientific and Cultural Organization (UNESCO).
• World Heritage Sites are designated by UNESCO for having cultural, historical, scientific or other forms
of significance.
o UNESCO seeks to encourage the identification, protection and preservation of cultural and natural
heritage around the world considered to be of outstanding value to humanity.
• The World Heritage Sites list is maintained by the international World Heritage Program administered by
the UNESCO World Heritage Committee, composed of 21 "states parties" that are elected by their
General Assembly. Sites are demarcated by UNESCO as protected zones.
• Presently, there are 38 World Heritage Sites located in India. Some of them are:
o Mountain Railways of India
§ They represent a collective listing of the Darjeeling Himalayan Railway, the Nilgiri Mountain
Railway and the Kalka-Shimla Railway under the UNESCO World Heritage Site.
§ They are recognised for being "outstanding examples of bold, ingenious engineering solutions for
the problem of establishing an effective rail link through a rugged, mountainous terrain".
§ Hence option 3 is correct.
o Elephanta Caves
§ They are a network of sculpted caves located on Elephanta Island.
§ It consists of two groups of caves — five Hindu caves & two Buddhist caves.
§ The caves are hewn from solid basalt rock and designated as World Heritage Site to preserve the
artwork.
§ Hence option 1 is correct.
• Six sites, including the Ganga ghats in Varanasi, temples of Kancheepuram in Tamil Nadu and the
Satpura Tiger Reserve in Madhya Pradesh, have been added to India’s tentative list of UNESCO world
heritage sites. Hence, options 2 and 4 are not correct.
• Six of the nine sites submitted by the Archaeological Survey of India had been accepted by UNESCO for
inclusion in the tentative list, which is a requirement before the final nomination of any site.
• The final nomination for a site could be submitted after the proposal had been on the tentative list for a
year.

Q 82.A
• Recent Context: Government has released 8th instalment of financial benefit under PM-KISAN to infuse
liquidity at the grassroots (village) level and will help to boost rural demand.
• Pradhan Mantri Kisan Samman Nidhi (PM-KISAN) is a Central Sector scheme with 100% funding
from Government of India. The Scheme is effective from 1.12.2018. Hence statement 1 is correct.
o The scheme was started with a view to augment the income of the farmers by providing income
support to all landholding farmers’ families across the country, to enable them to take care of
expenses related to agriculture and allied activities as well as domestic needs.
o Under the Scheme an amount of Rs.6000/- per year is transferred in three 4-monthly
installments of Rs.2000/- directly into the bank accounts of the farmers, subject to certain
exclusion criteria relating to higher income status.
§ All installments falling due on or after 1st December, 2019 are being paid only on the basis of
Aadhaar authenticated bank data of beneficiaries to ensure genuine beneficiaries and avoid
duplicacy in payments, except in respect of the States of Assam and Meghalaya besides the
UTs of Jammu & Kashmir and Ladakh which are exempted. Hence statement 2 is not
correct.
o The Scheme initially provided income support to all Small and Marginal Farmers’ families across the
country, holding cultivable land up to 2 hectares. Its ambit was later expanded w.e.f. 01.06.2019 to
cover all farmer families in the country irrespective of the size of their land holdings.
o Definition of family for the Scheme is husband, wife and minor children.
o The entire responsibility of identification of beneficiary farmer families rests with the State / UT
Governments.
o The fund is directly transferred to the bank accounts of the beneficiaries.
o Farmers covered under the Exclusion Criteria of the Operational Guidelines are not eligible for the
benefit of the Scheme.

31 www.visionias.in ©Vision IAS

Google it:- https://www.pdfnotes.co


https://www.pdfnotes.co << Download From >> https://
www.pdfnotes.co
§ Exclusions from PM-Kisan include institutional land holders, farmer families holding
constitutional posts, serving or retired officers and employees of state/central government as well
as PSUs and government autonomous bodies. Hence statement 3 is correct.
o For enrollment, the farmer is required to approach the local patwari / revenue officer / Nodal Officer
(PM-Kisan) nominated by the State Government.
o The Common Service Centres (CSCs) have also been authorized to do registration of the farmers for
the Scheme upon payment of fees.
o Farmers can also do their self-registration through the Farmers Corner on the portal. Farmers
can also edit their names in PM-Kisan database as per their Aadhaar database / card through the
Farmers Corner in the portal.

Q 83.A
• The footloose industry is a general term for an industry that can be placed and located at any location
without effect from factors such as resources or transport. Therefore these industries enjoy a relatively
free choice of location. Diamonds and computer chips are some examples of footloose industries. Hence,
statement 1 is correct.
• Footloose industries can be located in a wide variety of places. They are not dependent on any specific
raw material, weight loss, or otherwise. They largely depend on component parts which can be obtained
anywhere. Hence, access to raw material is not an important factor influencing the location of
footloose industries. The important factor in their location is accessibility by road network.
• They produce in small quantities and also employ a small labor force. These are generally not polluting
industries. Hence, statement 2 is not correct.
• The reason for the growth of footloose industries lies in the rapid development of highly sophisticated
products requiring a great deal of scientific research and development. Footloose industries enable quick
product improvement of their products to suit the market demand.

Q 84.A
• The population of India according to their economic status is divided into three groups, namely; main
workers, marginal workers, and non-workers. It is observed that in India, the proportion of workers
(both main and marginal) is only 39.8 percent (Census 2011) leaving a vast majority of about 60
percent as non-workers. It is understood that, in the context of a country like India, the work
participation rate tends to be higher in the areas of lower levels of economic development since a
number of manual workers are needed to perform the subsistence or near subsistence economic
activities. Hence statement 2 is not correct.
• The occupational composition of India’s population shows a large proportion of primary sector workers
compared to secondary and tertiary sectors. As far as the occupation of the country’s male and female
population is concerned, male workers outnumber female workers in all three sectors. The number of
female workers is relatively high in the primary sector, though in recent years there has been some
improvement in the work participation of women in secondary and tertiary sectors. Hence statement 1 is
correct.

Q 85.A
• Mica in India is produced in Jharkhand, Andhra Pradesh, Telangana and Rajasthan followed by Tamil
Nadu, West Bengal, and Madhya Pradesh.
• In Andhra Pradesh, the Nellore district produces the best quality mica.
• In Rajasthan, the mica belt extends for about 320 km from Jaipur to Bhilwara and around Udaipur.
• Mica deposits also occur in Mysuru and Hasan districts of Karnataka, Coimbatore, Tiruchirapalli,
Madurai and Kanniyakumari in Tamil Nadu, Alleppey in Kerala, Ratnagiri in Maharashtra, Purulia, and
Bankura in West Bengal.
• Mica is a non-metallic mineral mainly used in the electrical and electronic industries. It can be split into
very thin sheets which are tough and flexible. Hence option (a) is the correct answer.

Q 86.B
• Shale gas is a natural gas produced from shale, a type of sedimentary rock. Due to constant
announcements of shale gas recoverable reserves, as well as drilling in Central Asia, South America, and
Africa, deepwater drilling, estimates are undergoing frequent updates, mostly increasing.
• Unlike conventional hydrocarbons that can be extracted from permeable rocks easily, shale gas is
trapped under low permeable rocks. It requires a mixture of ‘pressurized water, chemicals, and sand’
32 www.visionias.in ©Vision IAS

Google it:- https://www.pdfnotes.co


https://www.pdfnotes.co << Download From >> https://
www.pdfnotes.co
(shale fluid) to break low permeable rocks and have the access to the shale gas reserves. Hence,
statement 1 is not correct.
• Around 5 to 9 million litres of water is used per attempt of extraction (fracturing) activity. These
(fracturing) activities are likely to deplete water sources and cause pollution due to the disposal of
flow back (produced) water. Flowback water is the fluid that returns to the surface when shale fluid is
injected underground at a high pressure to fracture the rock. Hence, statement 2 is correct.
• China is estimated to have the world's largest shale gas reserves followed by the USA, Argentina,
Mexico, South Africa, Australia, and Canada. Hence, statement 3 is not correct.
• The United States and Canada are the major producers of commercially viable natural gas from shale
formations in the world, even though about a dozen other countries have conducted exploratory test wells.
China is the only nation outside of North America that has registered commercially viable production of
shale gas.

Q 87.D
• There are six religions in India that have been awarded "National minority" status—Muslims,
Christians, Sikhs, Jains, Buddhists, and Zoroastrians (Parsis).
• Hindus are distributed as a major group in many states (ranging from 70 - 90 percent and above) except
the districts of states along Indo- Bangladesh border, Indo-Pak border, Jammu & Kashmir, Hill States of
North-East and in scattered areas of Deccan Plateau and Ganga Plain.
• Muslims, the largest religious minority, are concentrated in Jammu & Kashmir, certain districts of West
Bengal and Kerala, many districts of Uttar Pradesh, in and around Delhi, and in Lakshadweep. They form
the majority in Kashmir valley and Lakshadweep. Hence statement 2 is correct.
• The Christian population is distributed mostly in rural areas of the country. The main concentration is
observed along the Western coast around Goa, Kerala, and also in the hill states of Meghalaya, Mizoram,
Nagaland, Chotanagpur area, and Hills of Manipur.
• Jains and Buddhists, the smallest religious groups in India have their concentration only in selected areas
of the country. Jains have a major concentration in the urban areas of Rajasthan, Gujarat, and
Maharashtra, while the Buddhists are concentrated mostly in Maharashtra. The other areas of the Buddhist
majority are Sikkim, Arunachal Pradesh, Ladakh in Jammu & Kashmir, Tripura, and Lahul and Spiti in
Himachal Pradesh.
• Religious communities of India, Census 2011
Religious Group Percentage of Total
Hindus 79.8
Muslims 14.2
Christians 2.3
Sikhs 1.7
Buddhists 0.7
Jains 0.4
Other Religions and Persuasions 0.7
• Hence statements 1 and 3 are correct.

Q 88.B
• Demographic Transition Theory tells us that the population of any region changes from high births and
high deaths to low births and low deaths as society progresses from rural agrarian and illiterate to urban
industrial and literate society.
• It has four stages.
• The first stage has high fertility and high mortality because people reproduce more to compensate
for the deaths due to epidemics and variable food supply. The population growth is slow and most
of the people are engaged in agriculture where large families are an asset. Life expectancy is low,
people are mostly illiterate and have low levels of technology.
• Thus though the fertility rate is high mortality rate is also high and there is no scope for population
expansion in this stage. Hence statement 1 is not correct.
• In the second stage, fertility remains high but mortality falls rapidly due to improvements in health
facilities and other socio-economic improvements. As a result, there is an expansion of the population.
• In the third stage, fertility falls rapidly but mortality falls slowly which leads to slow population growth
compared to the second stage.

33 www.visionias.in ©Vision IAS

Google it:- https://www.pdfnotes.co


https://www.pdfnotes.co << Download From >> https://
www.pdfnotes.co
• In the last stage, both fertility and mortality decline considerably. The population is either stable or
grows slowly or may even decline. The population becomes urbanized, literate and has high
technical know-how, and deliberately controls the family size. Hence statement 2 is correct.
• Developed countries like the USA, Canada, western European nations are believed to be in the last
stage.

Q 89.C
• Recent Context: Tianwen-1 entered Mars orbit in February 2021 and Zhurong (its rover) landed on May
14, 2021.
• Tianwen-1 ("questions to heaven," or "questioning the heavens") is China's first Mars mission,
consisting of an orbiter and a rover named Zhurong. Zhurong is named after a god of fire for a planet
known in Chinese as the planet of fire. Hence, both statements 1 and 2 are correct.
• Zhurong rover landed on a site on a vast last plane known as Utopia Planitia that is more than 2000 km
away from Jezero Crater where the US rover Perseverance landed. Zhurong rover, will provide first-hand
materials for research on the planet's space environment, surface topography, and soil structure.
• So far only the Soviet Union and the United States had previously carried out a successful landing on
Mars. Thus China became the third country to have soft landed successfully on the surface of Mars.

Q 90.C
• The Talcher coal mines are a 61.83 million tonne per annum (MTPA) network of mines in Angul,
Odisha, India, managed by Mahanadi Coalfields Limited, a subsidiary of Coal India. Hence pair 1 is
correctly matched.
• The coal from Talcher is supplied to southern and western Indian states for power generation. The central
government and the Odisha government are now working towards developing rail lines between Talcher
with Paradip port in Odisha to boost its transportation to other states via sea-route.
• Rampur is another important coal mine in the state of Odisha.
• The Korba coal mines are a 83 million ton-per-annum (MTPA) network of mines on the Korba
coalfield in Chhattisgarh state, India. Hence pair 2 is not correctly matched.
• The 83 million ton-per-annum Korba coal mine are owned by South Eastern Coalfields Limited (a
subsidiary of state-owned Coal India).
• Neyveli is a mining and power generation township in Cuddalore district of Tamil Nadu. Hence
pair 3 is correctly matched. A large portion of Thermal power generated in Tamil Nadu comes from
Neyveli.
• It is operated by Neyveli Lignite Corporation (NCL) India Limited.
34 www.visionias.in ©Vision IAS

Google it:- https://www.pdfnotes.co


https://www.pdfnotes.co << Download From >> https://
www.pdfnotes.co
• Tertiary coals occur in Assam, Arunachal Pradesh, Meghalaya and Nagaland. The Namphuk coal mines
are the only coal mines (in changlang district) of Arunachal Pradesh. Hence pair 4 is not correctly
matched.
• The Supreme Court lifted the ban on mining in the region in 2019, after which, coal mining was resumed
after seven years.

Q 91.A
• Coal is a one of the important minerals which is mainly used in the generation of thermal power and
smelting of iron ore. Coal occurs in rock sequences mainly of two geological ages, namely Gondwana
and tertiary deposits.
• The older Gondwana formations of peninsular plateau makes up to 98% of the total reserves in
India. Hence statement 2 is correct.
35 www.visionias.in ©Vision IAS

Google it:- https://www.pdfnotes.co


https://www.pdfnotes.co << Download From >> https://
www.pdfnotes.co
• It contains superior quality coal which is free from moisture. The most important Gondwana coal fields of
India are located in Damodar Valley.
• The coal occurrences in India are mainly distributed along the present day river valleys. Over 97 per cent
of coal reserves occur in the valleys of Damodar, Sone, Mahanadi and Godavari. Hence statement 1
is correct.
• Jharia (Jharkhand) is the largest coal field followed by Raniganj (West Bengal). Hence statement 3
is not correct.
• Other Major coal producing states of India are Madhya Pradesh, Odisha, Chhattisgarh, Andhra Pradesh
and Telangana.
• Tertiary coals occur in Assam, Arunachal Pradesh, Meghalaya and Nagaland.
• The Brown coal or lignite occur in the coastal areas of Tamil Nadu, Puducherry, Gujarat and Jammu
and Kashmir.

Q 92.D
• Shifting Cultivation
o Shifting cultivation, locally called 'Jhum', is a widely practiced system of crop cultivation among the
indigenous communities of Northeast India. Slash and burn cultivation is called ‘Podu Chaso’ in
the tribal regions of Orissa. The practice, also known as slash-and-burn agriculture, is when farmers
clear land by slashing vegetation and burning forests and woodlands to create clear land for
agricultural purposes.
o This provides a very easy and very fast method of the preparation of the land for agriculture. The bush
and the weeds can be removed easily. The burning of waste materials provides needed nutrients for
the cultivation. It gives a family its food, fodder, fuel, livelihood and is closely linked to its identity.
Because of the cutting of forests and trees, this practice leads to soil erosion and may also affect the
course of rivers.
o Different local names used for shifting cultivation in different countries:
§ Ladang: Malaysia
§ Taungya: Burma
§ Tamrai: Thailand
§ Caingin: Philippines
§ Humah: Java
§ Chena: Sri Lanka
§ Milpa: Africa, Central America, and Mexico
• Hence option (d) is the correct answer.

Q 93.B
• Non-conventional energy sources are sustainable energy sources that are continuously replenished
by natural processes.
• These cannot be exhausted easily and can be generated constantly so that they can be used again and
again.
• These energy sources are more equitably distributed and environmentally friendly.
• Natural gas is a conventional source of energy as it is not renewable. It is a fossil energy source
formed beneath the earth's surface. Hence option 1 is not correct.
• It is called natural gas because it is formed naturally however, it is available in limited amounts.
• Bio-energy refers to energy derived from biological products which include agricultural residues,
municipal, industrial, and other wastes. It is a non-conventional source of energy. Hence option 2 is
correct.
• Bioenergy is a potential source of energy conversion. It can be converted into electrical energy, heat
energy, or gas for cooking.
• It will also process the waste and garbage and produce energy.
• Ocean currents are the storehouse of infinite energy. Tides and Waves are important sources of non-
conventional source of energy in the form of Tidal and Wave energy. Hence option 3 is correct.
• India has great potential for tidal and waves energy however, it is not utilized fully yet.
• Firewood is the wood that is burned and used as fuel. Since the green cover is also limited and not
replenished naturally, therefore, it is a conventional source of energy. Hence option 4 is not correct.

36 www.visionias.in ©Vision IAS

Google it:- https://www.pdfnotes.co


https://www.pdfnotes.co << Download From >> https://
www.pdfnotes.co
Q 94.A
• Secondary activities are those activities that involve the manufacturing of finished goods by processing
the raw materials that are found in nature.
• Secondary activities add value to natural resources by transforming raw materials into valuable
products. Secondary activities are concerned with manufacturing, processing, and construction
(infrastructure) industries.
• Cotton in the boll has limited use but after it is transformed into yarn, becomes more valuable and can be
used for making clothes. The cotton textile industry involves spinning, weaving, and finishing fabrics.
• Iron ore, cannot be used; directly from the mines, but after being converted into steel it gets its value and
can be used for making many valuable machines, tools, etc. Iron is extracted from iron ore by smelting in
a blast furnace with carbon (coke) and limestone.
• Agro-processing (food processing) involves the processing of raw materials from the field and the farm
into finished products for rural and urban markets. Majoragro-processing industries are food processing,
sugar, pickles, fruit juices, beverages (tea, coffee, and cocoa), spices and oils fats and textiles (cotton, jute,
silk), rubber, etc.
• The mining of Iron ore is a primary activity. Primary activities are directly dependent on the
environment as these refer to the utilization of the earth's resources such as land, water, vegetation,
building materials, and minerals.
• Hence, option (a) is the correct answer.

Q 95.C
• Recent context: National Aeronautics and Space Administration (NASA) aims to launch a new
mission to the dark side of the Moon. This mission Volatiles Investigating Polar Exploration Rover
(VIPER) will be launched on the Moon in search of critical water molecules and other sustaining
minerals. The mobile robot (rover) will be supplementary to the Artemis programme.
o The Volatiles Investigating Polar Exploration Rover (VIPER), the robot will map resources at the
lunar South Pole that could one day be harvested for long-term human exploration at the Moon. The
mission is planned for 2023. Hence, statement 1 is correct.
• The VIPER mission will search for the concentration water ice both over and under the surface of
the Moon, that could eventually be harvested to sustain human exploration on the Moon and the
Mars. The data received from VIPER has the potential to aid the scientists in determining precise
locations and concentrations of ice on the Moon and will help NASA in evaluating the environment and
potential resources at the lunar south pole in preparation for Artemis astronauts. Hence, statement 2 is
correct.
• During its 100-Earth-day mission, the VIPER rover will roam several miles and use its four science
instruments to sample various soil environments. The rover will explore lunar craters using a specialised
set of wheels and a suspension system to cover a variety of inclines and soil types.It will operate in the
permanently shadowed regions of the Moon that haven’t seen sunlight in billions of years and are some of
the coldest spots in the solar system.

Q 96.B
• India is endowed with fairly abundant resources of iron ore. It has the largest reserve of iron ore in
Asia. Haematite and magnetite two types of ores found in India. About 95 percent of the total reserves of
iron ore are located in the States of Odisha, Jharkhand, Chhattisgarh, Karnataka, Goa, Telangana, Andhra
Pradesh, and Tamil Nadu.
• The districts of Chandrapur, Bhandara, and Ratnagiri in Maharashtra produce iron ore.
• Durg is an important mine in Chhattisgarh. It forms the Durg-Bastar-Chandrapur belt of iron ore-
producing areas. This belt is in Maharashtra and Madhya Pradesh.
• Jharkhand has some of the oldest iron ore mines and most of the iron and steel plants are located around
them. Most of the important mines such as Noamundi and Gua are located in Poorbi and Pashchimi
Singhbhum districts.
• In Odisha, iron ore occurs in a series of hill ranges in Sundergarh, Mayurbhanj, and Jhar. The
important mines are Gurumahisani, Sulaipet, Badampahar (Mayurbhaj), Kiruburu (Kendujhar) and Bonai
(Sundergarh).

37 www.visionias.in ©Vision IAS

Google it:- https://www.pdfnotes.co


https://www.pdfnotes.co << Download From >> https://
www.pdfnotes.co

• Hence option (b) is the correct answer.

Q 97.A
• The clustered rural settlement is a compact or closely built-up area of houses. In this type of village, the
general living area is distinct and separated from the surrounding farms, barns, and pastures.
• Such settlements are generally found in fertile alluvial plains and in the northeastern states.
• Sometimes, people live in compact villages for security or defence reasons, such as in the Bundelkhand
region of central India and in Nagaland.
• In Rajasthan, scarcity of water has necessitated compact settlement for maximum utilization of available
water resources.
• The extremely fragmented nature of the terrain and land resource base of habitable areas results in
Dispersed settlements not clustered rural settlements.
• Hence option (a) is the correct answer.

38 www.visionias.in ©Vision IAS

Google it:- https://www.pdfnotes.co


https://www.pdfnotes.co << Download From >> https://
www.pdfnotes.co
Q 98.D
• Most of the metallic minerals in India occur in the peninsular plateau region in the old crystalline
rocks. Minerals are generally concentrated in three broad belts in India. There may be some sporadic
occurrences here and there in isolated pockets.
• These belts are The North-Eastern Mineral belt, The South-Western Mineral belt, and The North-
Western Mineral belt.
• The Himalayan belt is another mineral belt where copper, lead, zinc, cobalt, and tungsten are known to
occur. They occur in both the eastern and western parts.
• The southwestern belt covers the southwestern plateau region. It extends over Karnataka, Goa, and
contiguous Tamil Nadu uplands and Kerala.
• Bauxite and ferrous metals are abundant in this belt. There is also high-grade iron ore, manganese,
and limestone. Except for Neyveli lignite, this area is devoid of any coal reserves.
• This belt does not have as diversified mineral deposits as the northeastern belt. It contains deposits of
iron ore, ilmenite, zircon, monazite sands, garnet, china-clay, bauxite, mica, limestone, and
soapstone.
• Kerala has deposits of monazite and thorium, bauxite clay. Goa has iron ore deposits.
• Hence option (d) is the correct answer.

Q 99.C
• India is a land of linguistic diversity. In the context of modern India, there are about 22 scheduled
languages and a number of non-scheduled languages. The speakers of major Indian languages belong to
four language families, which have their sub-families and branches or groups. They are
o Indo-European(Aryan)e.g. Hindi, Bengali, Marathi, Urdu, Pahari, Oriya, Assamese
o Dravidian, e.g.: Telugu, Tamil, Kannada, Malayalam,Malto,Kui,Toda,Gadaba·
o Austric e.g.: Munda or Kol Group, Mon-Khmer groups like Khasi and Nicobarese.
o Sino-Tibetan families(Kirata languages) e.g.:Tibetian, Sikkimese, Bhotia, Ladhaki, Abor, Dafla,
Naga, etc.,
• About three-fourths of the Indian population speaks different languages of the Aryan family. Next
comes the Dravidian family which is spoken by more than 20 percent of the Indian population. The
Austric and the Sino-Tibetan languages are spoken by a small percentage of people i.e., 1.38% and
0.85% respectively.
• Additional Information:
• Indo-Aryan: It comprises all the principal languages of northern and western India such as Hindi,
Bengali, Marathi, Gujarati, Punjabi, Sindhi, Rajasthani, Assamese, Oriya, Pahari, Bihari, Kashmiri, Urdu,
and Sanskrit.
• Dravidian: The Northern group consists of three languages i.e. Brahui, Malto, and Kudukh. The Central
group consists of eleven languages viz., Gondi, Khond, Kui, Manda, Parji, Gadaba, Kolami, Pengo, Naiki,
Kuvi, and Telugu. The southern group consists of seven languages viz., Kannada, Tamil, Malayalam,
Tulu, Kodagu, Toda, and Kota.
• Sino-Tibetan: These languages are considered to be older than the Indo-Aryan languages and are referred
to in the oldest Sanskrit literature as Kiratas. The Tibeto-Burman languages are divided into four broad
groups:
o Tibetan: Sikkimese, Bhotia, Balti, Sherpa, Lahuli, and Ladakhi
o Himalayan: Kanauri and Limbu
o North-Assam: Abor (Adi), Miri, Aka, Dafla, and Mishmi
o Assam-Burmese: It is again sub-divided into four main sub-groups, viz. Kuki-Chin, Mikir, Bodo and
Naga.
• The Austric languages of India belong to the Austro-Asiatic sub-family, which are represented by
languages of the Munda or Kol Group, spoken in the central, eastern, and north-eastern India and
languages of the Mon-Khmer group like Khasi and Nicobarese.
• Hence option (c) is the correct answer.

39 www.visionias.in ©Vision IAS

Google it:- https://www.pdfnotes.co


https://www.pdfnotes.co << Download From >> https://
www.pdfnotes.co
Q 100.B
• The Border Road Organisation (BRO) was established in May 1960 for accelerating economic
development and strengthening defence preparedness through the rapid and coordinated improvement of
strategically important roads along the northern and north-eastern boundary of the country. It works under
the administrative control of the Ministry of Defence.
• Konkan Railway was constructed in 1998. It is a 760-km long rail route connecting Roha in Maharashtra
to Mangalore in Karnataka. It is considered an engineering marvel. It crosses 146 rivers, streams, nearly
2000 bridges, and 91 tunnels.
• For the development, maintenance, and regulation of national waterways for shipping and navigation in
the country, the Inland Waterways Authority was set up in 1986. The Authority primarily undertakes
projects for the development and maintenance of IWT infrastructure on national waterways through grant
received from the Ministry of Shipping. The head office of the Authority is at Noida.
• Oil India Limited (OIL) under the administrative set up of the Ministry of Petroleum and Natural Gas is
engaged in the exploration, production, and transportation of crude oil and natural gas. It was incorporated
in 1959 as a company. The corporation is a Navratna with its corporate offices in Noida in the New Delhi-
NCR region.
• Hence option (b) is the correct answer.

Copyright © by Vision IAS


All rights are reserved. No part of this document may be reproduced, stored in a retrieval system or
transmitted in any form or by any means, electronic, mechanical, photocopying, recording or
otherwise, without prior permission of Vision IAS.

40 www.visionias.in ©Vision IAS

Google it:- https://www.pdfnotes.co

You might also like